Socratic Grilling

Imagine an kid in school first hearing about germ theory. The conversation might go something like this:

Teacher: Many diseases like the common cold are spread by germs, when one infected person contacts another.

Student: But I got a cold a few weeks ago, and I never touch anyone except my family members. And none of them were sick.

Teacher: You don’t need to actually touch someone. Sometimes it can spread through mucus droplets in the air.

Student: And one time I was camping in the woods for a month, and then I got a cold, even though I hadn’t been around anybody.

Teacher: If it was spring, you might have gotten allergies. Allergies can feel a lot like a cold, but they aren’t spread by germs.

Student: It was fall.

Teacher: Then maybe it was an unusual allergy, or some other condition.

Student: Hey, wait. If germs are spread from person to person on touch, why doesn’t the government just mandate one week when nobody is allowed to touch anyone else? Then all the germs will die and we’ll never have to worry about germs again.

Teacher: That’s a good question. A lot of germs have what’s called reservoirs in the environment, where they live when they’re not infecting humans. Even if the government tried your plan, probably most sicknesses would come back from their reservoirs.

Student: I’ve never seen a germ reservoir. Where are they?

Teacher: They’re not literal reservoirs like a water reservoir, that’s just what we call it when germs live in bats or raccoons or something.

Notice a few things about this conversation.

First, it’s really good that it happened. The student was clearly confused at many points. First, he had direct evidence that seemed to contradict the teacher’s claims that germs only spread by touch. Second, he had a sort of efficient-market-style confusion: germ theory seems to imply an easy way to eliminate all sicknesses forever, so why hasn’t someone picked this low-hanging fruit? Third, he was confused by an awkward term – he thought germ reservoirs were lakes full of germs in the hills somewhere.

All of these confusions are totally understandable. In fact, they’re a really good sign he’s paying attention – that he’s trying to figure out what germ theory really means and how it interacts with the rest of his worldview. They’re the direct opposite of guessing the teacher’s password. One day this kid is going to be an amazing scientist.

Second, to a hostile observer, it would sound like the student was challenging the teacher. Every time the teacher tried to explain germ theory, the student “pounced” on a supposed inconsistency. When the teacher tried to explain the inconsistency, the student challenged her explanations. At times he almost seems to be mocking the teacher. Without contextual clues – and without an appreciation for how confused young kids can be sometimes – it could sound like this kid is an arrogant know-it-all who thinks he’s checkmated biologists and proven that germ theory can’t possibly be true. Or that he thinks that he, a mere schoolchild, can come up with a novel way to end all sickness forever that nobody else ever thought of.

And the thesis of this post is that you must never, ever say that. Saying that is so bad. Smack down that student once, say “I think I know more about germ theory than you do”, make him feel like he challenged your authority and that’s bad – and the best case scenario is he will never ask questions to resolve his confusion again. The worst case scenario is that he stops feeling the confusion entirely, or stops thinking of forcing things to fit together and make sense as a desirable goal to have.

One of the most important rationalist skills is “noticing your confusion”. But that depends on an even more important proto-skill of wanting things to make sense. If you lose that skill – if it stops bothering you and seeming like a problem when things don’t make sense to you – you will never notice your confusion and you will never become a good scientist or a good anything-else-that-requires-independent-thought. And interpreting an attempt to explore dissonance as a status grab that needs to be knocked down is absolutely fatal for that skill. Instead, you need to think of it as Socratic grilling – like Socratic questioning, but a little harsher and more confrontational in order to get to the point more quickly.

Tolerating this is harder than it sounds. Most people can stay helpful for one or two iterations. But most people are bad at explaining things, so one or two iterations isn’t always enough I’ve had times when I need five or ten question-answer rounds with a teacher in order to understand what they’re telling me. The process sounds a lot like “The thing you just said is obviously wrong”…”no, that explanation you gave doesn’t make sense, you’re still obviously wrong”…”you keep saying the same thing over and over again, and it keeps being obviously wrong”…”no, that’s irrelevant to the point that’s bothering me”…”no, that’s also irrelevant, you keep saying an obviously wrong thing”…”Oh! That word means something totally different from what I thought it meant, now your statement makes total sense.”

But it’s harder even than that. Sometimes there is a vast inferential distance between you and the place where your teacher’s model makes sense, and you need to go through a process as laborious as converting a religious person to a materialist worldview (or vice versa) before the gap gets closed. The process of learning to really appreciate communism, or libertarianism, or whatever, coming from a diametrically opposed philosophy, looks a lot like dozens of questions about “but isn’t that an atrocity?” “wouldn’t this inevitably lead to dystopia?” and hearing what your interlocutor has to answer. It’s so, so tempting to round this off to them trying to gotcha you (as indeed sometimes it will be) and assume they’re not really committed to trying to understand.

With a good teacher, you can add enough disclaimers to your statements that they won’t get mad at you during this process: “I know I am but a mere student, and nowhere near smart enough to actually challenge you, so I’m sure I’m just misunderstanding this, but the thing you just said seems really confusing to me, and I’m not saying it’s not true, but I can’t figure out how it possibly could be true, which is my fault and not yours, but could you please try to explain it differently?” With a great teacher, all of this is assumed, and you don’t need the disclaimers, and you can just say “What? That makes no sense,” and expect the teacher to try again.

I think actual teachers get this right more often than not. What I’m concerned about is the self-appointed Internet anti-autodidact brigade. I occasionally get these people as commenters on my blog, but they’re more common in Reddit and places like that. Someone goes “that’s weird” or “if X was really true, wouldn’t that imply Y?” and gets hit with “You really think you’re smarter than everyone else? You really think a random person on the Internet has discovered a hole in X?” No, sometimes they’re just using Socratic grilling to expose the contradictions in their model and get somebody to resolve them.

I find this to be one of the most frustrating parts of writing this blog: how do I signal the things I still need to learn without the Arrogance Police descending on me? This is different from the case with a specific teacher – here there’s just me projecting my confusions out into the void to see if anyone answers. And every so often I do flatter myself that I’ve discovered a flaw in my betters’ reasoning, so it’s less obvious when I mean to imply that and when I don’t. Next time this happens I’m going to try just linking this post and seeing how it goes.

I think I strike a good balance, at the cost of endless annoying disclaimers that 95% of you don’t need. From now on, maybe I’ll just link this post.

This entry was posted in Uncategorized. Bookmark the permalink.

269 Responses to Socratic Grilling

  1. LGS says:

    I’m just asking questions, dear teacher, but I noticed I’m confused: how many people who merely “ask questions” in an arrogant manner do you think are really like the student, knowing their own ignorance and wanting to learn? And how many are truly mocking the teachings you have to offer even though they know nothing about the subject matter? What if the former group was only 1% of the questioners for the subject matter at hand? And regardless of all this, why do some rationalists have to act *so damn pretentious* when they ask these types of questions?

    Don’t take offense, I’m just asking.

    • melolontha says:

      Translation: I see everything through the lens of status and am more interested in keeping people in their place than helping them understand. If they’re sufficiently deferential then maybe I’ll deign to enlighten them, but I’ll probably just assume they’re acting in bad faith like everyone else.

      More seriously, if we’re talking about internet interactions, the ‘student’ probably should be sceptical of the ‘teacher’s’ authority by default. People bullshit and pose as more knowledgeable than they are all the time, and if they can’t/won’t answer a reasonable followup question, that is evidence against their credibility. From the genuine expert’s perspective this might feel ridiculous: I’m donating my expertise and this is how they react, with distrust instead of gratitude? But believing everything that you’re told in a sufficiently authoritative voice is a mug’s game.

      • LGS says:

        What’s this? A hostile “translation” which misrepresents me while I was merely asking questions? Sounds like you’re smacking me down, the thing Scott said to “never, ever” do. My innocent curiosity is getting crushed here! I was only trying to follow your advice and not trust people on the internet (like Scott) when they say stuff in an authoritative voice.

        (At the risk of killing the irony by explaining it: Scott’s advice of “never ever” smacking questioners down – not even when they’re being deliberately annoying/smug – is not good advice, and I made you violate it just now by being smug enough that you felt the need to smack me down.)

        • Skeptical Wolf says:

          Scott’s advice of “never ever” smacking questioners down – not even when they’re being deliberately annoying/smug – is not good advice

          As both a teacher and a student, I disagree. While “never ever” is obvious hyperbole, the underlying point is very valuable. This sort of mistake (taking offence at being questioned and/or seeing questions as challenges to their legitimate authority) is one of the most damaging ones a teacher can make, for exactly the reasons Scott describes. And yet, it’s also one of the most common teaching mistakes I’ve seen.

          If you see arrogance in every question, you may be treating the interaction as a status game yourself, rather than an opportunity for teaching or truth-seeking. If you find yourself upset over the lack of compliance/respect a question represents, then “prison guard” might be a better career to pursue than “teacher”.

          • acymetric says:

            I don’t think Scott’s point is supposed to be specific to classrooms. I get the impression that this was written in the “we are all the teacher when we are explaining things to people/asked questions by people” sense.

          • Simon_Jester says:

            As both a teacher and a student, I disagree. While “never ever” is obvious hyperbole, the underlying point is very valuable.

            The underlying point is very valuable, but there are people who act as if “never ever” is NOT obvious hyperbole and literally anyone should be prepared to answer any question at any time.

            Which leads to obvious failure states of its own when bad faith actors exist on the scene.

            The health of the student-teacher relationship is contingent on a lot of things… But one of them is the student’s willingness to accept the premise that the relationship even exists in the first place

            As a purely practical matter, one person cannot teach twenty unless the twenty observe certain rules of order. One person cannot even teach one unless there is a core acceptance that transfer of knowledge should be taking place.

            Believe it or not, some alleged ‘students’ (typically the ones who are themselves NOT genuine seekers after truth) will reject this premise and actively sabotage the learning environment, either for themselves or others. A mechanism for dealing with such cases is essential.

            And sometimes, the mechanism is “I cannot answer that question at this time.”

            Or “that question is outside the scope of the lesson, email me if you want an answer” and the student never does- because there won’t be time to answer the minimally necessary question if every question that can be conceived is asked first.

            Or “that is a very clever way to invalidate the analogy I made up to teach you, but did you get what I mean about this important concept I am trying to convey right now in this moment, because that was kiiind of important?”

            Or the original question was more like “why should we do this thing you told us to do” and the answer is “because I can’t teach you if you don’t proceed through some appropriate series of exercises and learning experiences, and my job is to select those sequences and guide you through them, and that means that we have a structure in place to ensure that you stop fussing about and let me do my job.

      • enkiv2 says:

        OK, but asking innocent-sounding questions that require a lot of complicated explanation in order to waste people’s time is a well-established tactic (up until about 15 years ago, this is what ‘trolling’ meant, before ‘trolling’ got generalized to include other behaviors like ‘flaming’ and ‘griefing’). Specifically seeking out people saying one thing in order to ask the same innocent-sounding questions over and over, with an aim to waste the time of people holding a particular position & thereby elevate the opposing position, is also a well-established tactic (called ‘sealioning’).

        Everybody who expresses opinions online in a public space (as opposed to a private or semi-private one like the timeline of a private facebook account) has encountered these tactics & developed some kind of mechanism to identify and defend against them. Assuming that everybody who asks you questions is genuinely confused is *also* a mug’s game for this reason (and this is why, for instance, FAQs were invented — even if the questions are genuine, answering the same ones over and over because somebody is unwilling or unable to search for the answers is a waste of time).

        One on one interactions with confused people, in an internet context, are usually not so fruitful — at best, you have made one person less confused, but other people with exactly the same confusion will probably not see the conversation! If that conversation can be performed and then mined for information in order to produce an FAQ or an essay, then the next time somebody seems to have the same confusion, you can just link them to the essay.

        Of course, there’s always the possibility of a double bind. Alice says something, and Bob sees it and finds it confusing, but thinks the odds are good that Alice is full of shit; Alice sees Bob’s response and thinks the odds are good that Bob is actually a sockpuppet for Carol, who has been pretending to have Bob’s confusion under assumed names in order to waste her time three times a day for six months. Alice sends Bob a link to an article that explains the subject along with all the various common misunderstandings, but Bob is unwilling to read six thousand words of content written by somebody who might be a crank, so he just asks questions instead. Alice treats this as confirmation that Bob is actually Carol and shuts the conversation down, and Bob treats Alice’s behavior as confirmation that she is a crank & cannot actually defend her points. They both leave the interaction convinced that the other party is not worth listening to in the future, totally unaware of gaps in their own understanding of the world that the other might be able to fill in.

        In this pattern, it doesn’t make a lot of sense (as a solution) for Alice to lend Bob more credibility, especially in situations where the Carols of the world are extremely active (such as hot-button political topics). It makes more sense for Bob to lend Alice more credibility and actually read the essay, because Alice is a broadcaster who has to deal with many Bobs and Carols at any given time and Bob has nothing better to do than to argue on the internet. Both Alice and Bob will be better served if Bob simply reads just enough of Alice’s essay to solve his confusion, or reads the whole essay & then asks any further questions, than if he continues to personally interrogate her in search of material already documented in the essay.

        • etaphy says:

          That attitude essentially boils down to a long term win for trolls, given that their purpose was to diminish the productiveness of conversations for kicks. If somebody sees a genuine(to their eyes at least) inconsistency in an essay-form explanation of a topic and is consequently unable to get an explanation about that inconsistency’s general impact on the question without being dismissed, the chances are that what they’re reading gets perceived as missionary and not explanatory work.
          If somebody is genuinely explaining a subject and not merely proselytizing or campaigning for status, they should be able to address simple inconsistencies.

          Turning conversations on complex subjects into “shut up and listen to the Knowledgeable Authority” simply generates an atmosphere of bad faith engagement on both ends. To one side it’ll appear like trolling(even though answering one such misperceived “troll” genuinely would likely introduce bonus clarity to the whole conversation), to the other – like condescending missionary work.

          • skybrian says:

            One trick is to ask a polite question, because at least this distinguishes you from simple trolls.

            And on the other end, to politely answer questions that is seems other folks may have. If there are too many or they are repetitive, consider writing a FAQ. If it’s a good question, it doesn’t matter if this particular person is a troll or not, so long as they don’t act like one.

            But some good questions require more research than we are willing to do in casual conversation, so the easy way out is just to be very willing to say you don’t know the answer, and that it is good practice to collect questions.

          • Simon_Jester says:

            I have encountered dozens of ‘non-simple’ trolls who are perfectly capable of maintaining basic civility while asking pointless questions they don’t actually care about the answers to.

            At least, so long as you define ‘civil’ to include things like ‘so why should we not brutally murder those guys over there’ and ‘I think everything about your life that causes you problems is a figment of your own imagination’ and ‘this evidence I cherrypicked for the 37th time clearly shows that you’re all wrong, please make an hour long effort post to refute what I spent five minutes half-assing.’

            I’m not sure we can afford to have that be part of our definition of ‘civility’ if we want an Internet where people are willing to answer civil questions. There is only so much time, and eventually you hit a “floor” where the percentage of time wasted talking to superficially civil JAQ-offs outweighs any benefit to be had by educating the genuinely confused people.

        • Nietzsche says:

          Excellent comment. There is game theory in the background here, where both the student and the teacher are deciding how much to spend (time, attention, etc.) on their interaction. The teacher doesn’t want to overspend in case the student is trolling, and the student doesn’t want to overspend in case the teacher is a crank. But they are both prepared to pay something. Maybe there is a Nash equilibrium to be worked out by someone less lazy than me.

        • Nancy Lebovitz says:

          The earliest definition of trolling I’ve seen was deliberately posting errors in the hopes of conning someone into thinking you really didn’t know whatever so they’d correct you.

          When I think about it, this is a remarkably pathetic way of trying to feel superior, but it’s also evidence of a gentler era.

        • Ghatanathoah says:

          Is sealioning an actual thing though? Or is just something people are accused of when they ask other people to explain themselves in detail? Doesn’t it make just as much sense to think that supporters of a position are so ridiculously overconfident of its correctness that they can’t comprehend how it raises so many questions? That they don’t understand how large an inferential distance there can be between other people? That they often assume everyone else agrees with them about all the facts, but has decided to side with Team Evil instead of Team Good for some reason? Maybe that last one sounds paranoid, but it’s no more paranoid than thinking people who ask you questions must be trolls.

          The times I’ve been accused of sealioning and concern trolling have been when I’m asking questions to try to get people to articulate the deep-seated reasons behind their beliefs. Often they are so unused to questioning those that they don’t understand why I am trying to get them to explain something “obvious.”

          “Concern trolling” is another one I’m not sure actually exists. Which makes more sense, that people are asking questions to demoralize you, or that your views have some questionable elements? The people who talk about concern trolls tend to be people who are, again, so ridiculously sure of themselves and their rightness that they can’t even understand how someone could sincerely question them.

          I often wonder if sealioning and concern trolling are the “cow-tipping” of the online world. Something everyone has heard of, but no one has actually done.

          • Nornagest says:

            “Concern trolling” is another one I’m not sure actually exists. Which makes more sense, that people are asking questions to demoralize you, or that your views have some questionable elements?

            Polite disagreement is often expressed as accepting some of the premises of your partner’s worldview but gently questioning others, even if you actually disagree with the whole shebang (“I know we’ve got an aging population and an increasingly strained healthcare system, but it seems like it gives their relatives a lot of pain when we feed senior citizens to hyenas?”). Some of the people that throw around accusations of concern trolling are correctly identifying this pattern, but incorrectly reading malice into it. Others are pattern-matching actual concern to it.

          • Simon_Jester says:

            Is sealioning an actual thing though? Or is just something people are accused of when they ask other people to explain themselves in detail? Doesn’t it make just as much sense to think that supporters of a position are so ridiculously overconfident of its correctness that they can’t comprehend how it raises so many questions?

            Sealioning, as distinct from other related phenomena, isn’t about asking people to explain themselves in detail. It’s about a combination of:

            1) Making isolated or repetitive demands for rigor after a point has already been addressed, and:
            2) Being so persistent about a specific point that it prevents healthy or functional conversation about anything else.

            Remember that the original case of ‘sealioning’ included the sealion intruding on the speaker’s house, staying the night, and generally making normal life impossible. It is this sense of intrusion- of a person who does not want to be your community, entering a space your community needs in order to function, and shutting down the community until their own arbitrary demands for proof are satisfied- that caused “sealioning” to sprout a whole new term.

            “Concern trolling” is another one I’m not sure actually exists. Which makes more sense, that people are asking questions to demoralize you, or that your views have some questionable elements?

            The problem arises when people selectively pick out the questionable elements of your worldview and reject every proof you present, and gravitate towards dwelling on the questionable bits to the exclusion of all else while basically shrugging off attempts to justify them.

            At some point, again, this stops looking like a good faith case of someone willing to engage with or learn from others, and starts looking like “I am a committed warrior against your viewpoint who is looking for enemies to do battle with, and the way I choose to do battle is by repeating criticisms of your worldview that enable me to say “checkmate, liberals” under my breath as much as possible.

            The big problem is that it can be, experientially speaking, REALLY HARD to tell a genuinely confused person who is willing to learn apart from a malicious person who is never actually going to change their mind in response to evidence and is just there because fighting you is fun. This can lead to accidental radicalization, wasted time and mental energy, or both.

            Until and unless we clear our society up to the point where people stop being willing to sabotage ongoing discussions by feigning interest and then making boors of themselves, there’s just inherently a dilemma here

          • Spookykou says:

            @Simon_Jester

            entering a space your community needs in order to function, and shutting down the community

            I don’t really engage with the social internet much, besides SSC, so I honestly don’t have much of an impression of what online communities look like besides this one, but I feel like this community is not at risk of being shut down by sealioning. Is this a more realistic problem for other internet communities? Is it more of a problem for SSC than I think?

            As someone who has never seen the term sealioning before in the wild, it just feels like people not wanting to explain themselves, which is reasonable, but please just say that instead of otherizing your interlocutors.

          • enkiv2 says:

            Sealioning is the intersection of kiboing and trolling. It’s when you use modern search facilities to find people saying things you disagree with in order to waste your time. I’ll agree that sometimes people get accused of sealioning when they are really just accidentally running across posts on a particular topic they care about due to algorithmic recommendations, but I’ve seen genuine sealioning in the wild (in the form of people whose accounts are hundreds of near-identical replies to unrelated people they have no social connection to, with nothing else). And, as somebody with a particular subject I have deeply-held deeply-unpopular opinions about (hypertext), I’ve had to hold myself back consciously from engaging in sealioning behavior, so I know it’s something that a relatively well-adjusted person can fall into if they lack impulse control or self-awareness.

            I have also, in my youth, concern-trolled people — so I know it’s a thing that exists. I would get angry at someone, then realize that they were in the right but still be too angry to back down, so I would reframe my attacks as though they were actually attempts to help (half-convincing myself that they *were*). It’s exactly the kind of thing that arguing couples sometimes do.

            Most forms of internet griefing behavior are just scaled up versions of common dysfunctions of in-person conversation that people fall into when angry or afraid — it’s the equivalent of the extremely intense arguments couples often have, except you have them with a hundred strangers simultaneously. (Classic trolling is an interesting case because it’s *not* a common in-person thing & actually requires a calm demeanor, but it’s hardly new — it’s literally socratic dialogue.) I generally find it hard to believe that any given person who socializes online has never engaged in them (let alone seen them), and it’s more believable to me that somebody has done them without realizing it than that they have never encountered them at all, because (like most conversational gambits) they are only extremely rarely the product of conscious thought and long-term careful planning. They’re the kinds of things one says before realizing it.

          • Spookykou says:

            @enkiv2

            Google doesn’t know what kiboing is and neither do I unfortunately.

            This reminds me a bit of Scott’s post about paranoia not being a binary condition but existing on a spectrum and how two different people can have wildly different interpretations of the same interaction. For instance I think tone is important and for a long time the only thing I posted on SSC was tone policing comments(I did not think of myself as trolling, I actually wanted the tone of SSC to stay elevated). I am not sure I would catch most concern trolling, and I’m sure plenty of people have probably assumed that is what I was doing(I had to google concern trolling also, but I have seen it on here before).

          • Simon_Jester says:

            @Spookykou

            I don’t really engage with the social internet much, besides SSC, so I honestly don’t have much of an impression of what online communities look like besides this one, but I feel like this community is not at risk of being shut down by sealioning. Is this a more realistic problem for other internet communities? Is it more of a problem for SSC than I think?

            I would say ‘yes’ and ‘sort of.’

            To extend the analogy, suppose that sea lions naturally prey upon penguins. Any place sea lions are common, is a place penguins must avoid. Suppose, and here we make the analogy imperfect, that each species of sea lion preys on certain species of penguin. Creationist sea lions prey on evolutionist penguins, New Atheist sea lions prey on creationist penguins. Homophobic sea lions prey on gay penguins, and I don’t know if there are any gay sea lions preying on homophobic penguins or not but we can imagine it. Social justice sea lions prey on genetic determinist penguins, and genetic determinist sea lions prey on social justice penguins, whatever.

            In general, any given conversational space can contain a sea lion species or the corresponding penguin species that it preys upon, but not both.

            SSC has banned certain kinds of sealioning (you cannot, so far as I know, politely and repetitively tell a trans poster that their gender identity is an optical illusion and ‘ask’ them to explain why you’re wrong, or suchlike). It explicitly tolerates certain other kinds. Certain species of penguins appear here; others do not.

            The reason you do not see sealioning as a major threat in this community is because the predation dynamic has achieved fixation- the sea lions tolerated in this area have already eaten or driven off the corresponding penguin species.

            Someone seeking to found a new debating community, or someone who sees a penguin species as becoming endangered and wants to create a wildlife preserve for it, will have to think about the issue harder. So does someone who is themselves a penguin- a preferred prey species for some particular kind of sea lion.

            As someone who has never seen the term sealioning before in the wild, it just feels like people not wanting to explain themselves, which is reasonable, but please just say that instead of otherizing your interlocutors.

            It is reasonable to have a convenient term for a recurring pattern of behavior that recurs across multiple conversations and subjects. Just as we have a term for “no true Scotsman fallacy” or “weakmanning,” it is reasonable for us to have a term for “sealioning.”

            Namely, the repetitive and intrusive questioning of that which has already been established to some reasonable standard of good faith debating.

            Think about it. If a grown man intrudes upon a discussion of how to respond to the coronavirus epidemic with superficially civil insistence that the germ theory of disease be revisited and proven from the ground up, that man is behaving inappropriately. Something has clearly gone wrong. Even if in and of itself there is nothing wrong with the man’s questions, there are issues of time, place, and manner at work.

            The cluster of words for that behavior pattern includes “JAQ” and “sealioning.” These are mostly new words, because they were seldom needed before the Internet, because the Internet has made it much easier for random bozos to intrude upon conversations that cannot be held productively without agreeing on certain core principles.

          • Jiro says:

            Remember that the original case of ‘sealioning’ included the sealion intruding on the speaker’s house, staying the night, and generally making normal life impossible.

            The original case of “sealioning” was a cartoon that had the sealion intruding into someone’s house and refusing to leave, but was a bad faith accusation aimed at people who weren’t actually doing that (but who the left likes to treat as if they were.)

            If you’re going to pick a term, “concern trolling” usually works fine. Calling it “sealioning” implies that the case the term was coined for was real.

          • Spookykou says:

            @Simon_Jester

            I guess this does highlight a difference, to use your analogy, I feel like SSC is not a place with ‘penguins’ meaning prey. It is more like a community of Humboldt squid, where we are all ready to eat each-other at a moments notice(eating here meaning, engaging in an argument with). Civil squids though, at least as civil as squids can be.

          • The Nybbler says:

            Google doesn’t know what kiboing is and neither do I unfortunately.

            James “Kibo” Parry was (and is) a pre-WWW Usenet legend. Known for grepping (searching) all of Usenet for his nickname (“Kibo”) and replying wherever it came up.

            Thus the objection to sealioning is complaining that when you talk smack about people in public (even if you think they aren’t around), they show up and object.

          • enkiv2 says:

            Kibo is a figure on usenet famous for using an automated process to identify mentions of his name so that he could reply to them. As far as I can tell, he does not have a reputation as a jerk. However, as the ability to search newsgroups you don’t subscribe to became more common, more people would ‘kibo’ by searching for arbitrary terms of interest and jumping into threads without reading the long previous history of discussion in them — a major breach of netiquitte.

            The first notable sealion in usenet history is probably Sedar Argic, a bot that replied to every instance of the word ‘turkey’ with a long copypasta denying the armenian genocide — which became problematic around thanksgiving. When people complain about sealioning, this is the behavior they are describing (although the perpetrators are usually ostensibly-human and usually have marginally better context awareness, the unwillingness to seek full context is the source of annoyance).

          • Simon_Jester says:

            @Jiro

            The original case of “sealioning” was a cartoon that had the sealion intruding into someone’s house and refusing to leave, but was a bad faith accusation aimed at people who weren’t actually doing that (but who the left likes to treat as if they were.)

            The cartoon itself makes no such accusation; was it presented in a larger context on the original webpage that makes it into such an accusation?

            Also, it seems likely that “the left” sometimes makes accusations of the form “X is being done by group Y” that do not reduce to “every individual member of group Y is doing X.” For example, someone may be obnoxiously persistently demanding in their conversations in defense of [thing loathsome unto the left], occupying the left’s spaces in an unwelcome manner as the cartoon sealion occupies the cartoon house… Without every individual person doing that.

            Are you sure your argument isn’t equivalent to “the left accused sealions of intruding on their house and invasively trying to sustain a pointless debate, but here is a sealion who has never been in the left’s house, so clearly the accusation is false?”

            If you’re going to pick a term, “concern trolling” usually works fine. Calling it “sealioning” implies that the case the term was coined for was real.

            In short, would you mind expanding on the claim that the case the term was coined for wasn’t real?

            @Spookykou

            I guess this does highlight a difference, to use your analogy, I feel like SSC is not a place with ‘penguins’ meaning prey. It is more like a community of Humboldt squid, where we are all ready to eat each-other at a moments notice(eating here meaning, engaging in an argument with). Civil squids though, at least as civil as squids can be.

            Also, and this is one of the core reasons why leftists tend to be so conscious of behaviors like what I’ve called “sealioning…” Some individuals are unusually prone to problems brought about by certain repetitive arguments.

            For example, the gay guy probably can’t be at ease or feel much sense of community in an environment where people feel free to step up to him at any time and say “but seriously, why *isn’t* your thing about being attracted to other men just a degenerate lifestyle choice that will ultimately lead to the downfall of your civilization if promulgated it? Just asking.” The kind of environment where that can happen at any time, and importantly where the management doesn’t see a problem with it,

            And even if the gay guy is otherwise prepared to debate civilly and rationally on just about any other topic, he may understandably pack his bags and leave if faced with THAT topic in THAT manner. Because no one with any self-respect or pride can reasonably be expected to take such treatment on the chin over and over.

            So he may be a squid in your reckoning at times… but in that one specific way, he is a penguin being preyed upon by sea lions.

            If you look at the patterns of what kind of people are and aren’t here on the SSC comment threads, you may be inference be able to figure out which populations of rhetorical ‘prey’ and ‘predators’ avoid this space.

          • J Mann says:

            For what it’s worth, I think most of the complaints against sealioning are themselves close to gatekeeping, but I am reminded of Scott’s essay Against Interminable Arguments.

          • Jiro says:

            The cartoon itself makes no such accusation; was it presented in a larger context on the original webpage that makes it into such an accusation?

            What it is actually about is a blocked word here, but it starts with “gamerg” and ends with “ate”.

            And the context is not the webpage, but the web. The ants were a hot discussion topic at the time, it was understood by most people as referring to them, it was constantly used against them, and there really wasn’t anything else it could have meant.

            For example, the gay guy probably can’t be at ease or feel much sense of community in an environment where people feel free to step up to him at any time and say “but seriously, why *isn’t* your thing about being attracted to other men just a degenerate lifestyle choice

            On the other hand, if the “sealions” aren’t homophobes, but are instead people who have been accused of misogyny, and if you replace “speaking politely against homosexuality” with “speaking politely about why they are not misogynists”, it looks quite different.

          • J Mann says:

            The sealion guy wrote a hilarious clarification to his strip where conceded that if taken “as written,” it would be reasonable to infer that the humans in the strip were privileged bigots who drove fancy cars, wore fancy clothes, and were prejudiced against sea lions based on immutable characteristics of the species, but that as intended, he meant it to be people who disliked other people based on “certain behaviors.”

            I’ve always interpreted the “certain behaviors” to be promoting ethics in online journalism about virtual recreation, based on the timing of the cartoon and surrounding circumstances, but it’s never been spelled out.

          • Douglas Knight says:

            He also listed 4 pieces of press coverage, half of which mentioned ants, which is probably more relevant than what the author actually meant.

          • Namely, the repetitive and intrusive questioning of that which has already been established to some reasonable standard of good faith debating.

            The problem, especially online, where lots of different people with different backgrounds can get pulled into an argument, is that what “has already been established to some reasonable standard of good faith debating” is one of the things people disagree about.

            If I consider climate arguments, there are at least four separate claims involved and, as best I can tell, some people who honestly question each of them, along with quite a lot of people who assume that all four have been established.

        • J Mann says:

          I’m not a big fan of the concept of sealioning.

          I think it assumes more of the speaker’s intent than is reasonable – my model is that there are a few vandals who just enjoy stirring things up, but my guess is that people who otherwise engage in internet debate probably believe that they are correct and the people they’re debating are wrong.

          If you’re shouting Noam Chomsky or Charles Murray down so they can’t speak at a campus somewhere, then sure, but if you’re saying something substantive, even if it’s something the community thinks has already been fully debated, my guess is you probably mean it sincerely.

          As pointed out, the sealion sketch depends largely on the concept of property – the anti-sealion bigots in that sketch object to the sealion trespassing in their home to ask why they’re bigots.

          I don’t really buy that someone responding to my public tweet or showing up in my favorite blog’s comment threads is violating my property right. If anything, me posting to my friends and having them all agree creates a false sense that my views are public consensus, and it advertises that consensus publicly.

    • Deiseach says:

      Sometimes the chain of questions happens because the teacher is poor at explaining things. (I’ve been on that end). Sometimes it happens because the pupil is aware of their own stupidity and their need to be spoon-fed in order to really understand the topic (I have been on that end, too). Sometimes the pupil really is right and trying to get at “But do you not see the contradiction here, or is there a resolution that you are not telling me?” That too can happen: the teacher is not an expert on the topic and there is a resolution of the apparent contradiction but they are not at the level where they themselves learned it. A lot of passing on knowledge unhappily does go by rote ‘I learned this, now you have to learn it too, no questions’. Sometimes the pupil is being a smart alec and is not as clever as they think.

      Patience from both sides is a good thing.

      • Garrett says:

        > Sometimes the chain of questions happens because the teacher is poor at explaining things.

        I *am* that person. Which is why I no longer attempt to teach. It’s not that I don’t want to be good at it. It’s that I’ve noticed, repeatedly, that I’m really bad at it. And I haven’t found any good way to get better.

      • Randy M says:

        Sometimes the teacher wants to communicate the 101 level understanding, which is contradicted by the 401 model which is way beyond 95% of the intro class who are nonetheless perfectly happy to make monkey noises when someone owns the teacher.

        • Elementaldex says:

          I teach fencing and this is often aggravating. I state things as facts to beginners which are unquestionably false at high levels, but that will help them build a much stronger foundation if they treat them like facts now. I often get eyed or directly contradicted by people coming in with experience who really should not be working with beginners anyways (but our system has everyone together at the beginning so…). Its frustrating because going into long tangents about all the different ways things CAN be and how they change against higher level opponents or when you have a better ability to string actions together derails and makes the class worse. But its true that I am saying something which is false/wrong at the same time… I’ve tried lots of different ways to handle this and all of them have unfortunate failure modes.

          • Nick says:

            Reminder: there were two discussions a few months ago about the phenomenon of the lie-to-children: first, followup.

          • Elementaldex says:

            @Nick

            I have tried throwing in wiggle words to the effect that things change at higher levels, but that results in the most aggressive/adventurous people immediately discarding whatever foundational thing I’m trying to get them to do. Maybe it is better for everyone else and thus worth impairing those few? For context I only teach adults.

          • soreff says:

            Roughly speaking, what I do for a lot of cases is preface my statements with “Roughly speaking”. It is a reasonably quick way to warn that what I’m about to say is approximate and/or ignores corner cases.

      • Joshua says:

        Sometimes the chain of questions happens because the teacher is poor at explaining things.

        I offer a different frame, below: https://slatestarcodex.com/2020/03/06/socratic-grilling/#comment-861902

    • Saint Fiasco says:

      I think your confusion comes from the fact that you are thinking about probabilities but not enough about expected utilities.

      Even if most people who just ask questions are arrogant fools, you don’t lose much from answering honestly. Honest people who want to learn might be rare, but in that rare case you lose a lot if you answer harshly.

      Multiply a lot by a little, then substract a little multiplied by a lot, and you might end up better off just assuming good faith all the time.

      • Simon_Jester says:

        I think your confusion comes from the fact that you are thinking about probabilities but not enough about expected utilities.

        Even if most people who just ask questions are arrogant fools, you don’t lose much from answering honestly. Honest people who want to learn might be rare, but in that rare case you lose a lot if you answer harshly.

        Multiply a lot by a little, then substract a little multiplied by a lot, and you might end up better off just assuming good faith all the time.

        It depends heavily on the context; those weighting coefficients shift.

        Depending on who is asking what, of whom, and in what situation, you may see very sharp differences in whether or not it makes sense to assume good faith.

      • Jiro says:

        Even if most people who just ask questions are arrogant fools, you don’t lose much from answering honestly.

        This is subject to Goodhart’s Law. You don’t lose from answering honestly… until you actually start answering honestly, at which point you create bad incentives and answering honestly no longer works so well.

    • jimmy says:

      “how many people who merely “ask questions” in an arrogant manner do you think are really like the student, knowing their own ignorance and wanting to learn? ”

      More than you’d think, actually. People don’t always know how to do it in a non-arrogant and non-challenging way, and so they will challenge people that they expect to defeat their challenges so that they can see how to defeat these challenges in others — and I’ve had people admit explicitly that this is what they’re doing. It does come off as arrogant when you don’t see through the motivations, but when you look for the signs of motivated thinking in response to good arguments you make, it often isn’t actually there.

    • knzhou says:

      People here won’t like it, but this is the correct comment. I’ve had many conversations about physics with self-professed autodidacts online. The typical case is that I’ll explain that their question is a good one, that it was a serious consideration about 100 years ago, that it was decisively resolved by experiments about 75 years ago, and that this is all summarized in the opening chapters of various standard textbooks. A small amount thank me. A smaller amount ask smart followup questions. And everybody else goes on an unhinged rant about how I’m trying to steal their Nobel-winning idea, that they’re the new Galileo or Einstein, and I’m a hidebound reactionary trying to censor their brilliance, or whatever else.

      When the askers are actual kids, it always goes great, and we both walk away happy. But that’s not the typical case. After running into the worst case scenario a hundred times, I’ve just settled on some cheap heuristics. Is the asker over 35? Do they have a day job that’s in STEM but not in science or medicine (e.g. electrical engineer, coder)? Do they display any dislike of authority, at all? If at least two of these are true, a Galileo comparison is almost guaranteed. Call me closed-minded, but most physicists think I’m the open-minded fool, for even trying to talk to autodidacts at all.

      • The Nybbler says:

        Once again I’m going to gesture in the direction of the replication crisis. It is not true that the experts are always right or have all the answers. It is not even true that they haven’t overlooked things that a highly-intelligent fifth-grader would spot.

        • knzhou says:

          “Social scientist” isn’t a central example of “expert”. By my reckoning, a field that overlooks problems a child could see, by definition, does not have experts.

        • A1987dM says:

          But most of the people who will claim all experts have overlooked something they haven’t aren’t talking of stuff like power posing. They’re talking of stuff like special relativity, which has been shown way beyond any reasonable doubt to be correct at least to an excellent approximation within its domain of applicability.

        • Simon_Jester says:

          Yes, but there’s a gap between people who do this when talking about psychiatry (where the replication crisis is a nightmare), and people who do this when talking about physics (where you get a lot of cranks who find it personally insulting somehow that other people don’t think they can invent a perpetual motion machine).

          There’s also a ‘hubris gap.’ If I think the psychiatric community is wrong about something because it contradicts my lived experience and that of everyone I’ve talked to, I may be worth talking to. If I think the psychiatric community is wrong because I’m some kind of legendary genius who will show them all, I’m probably NOT worth talking to. For every person who really is a legendary genius and is boastful about it, there are a great many fools and madmen with delusions of grandeur.

      • zzzzort says:

        It’s a rather consistent pattern that the most insufferable groups of physicists (undergrads, autodidacts, emerita stuck in the 70’s) are the least likely to produce useful insights. This might make sense if asking insufferable questions is a tool for learning what other people have already figured out, but a bad tool for figuring out new things. Personally I think it’s mostly a bad habit that people need knowledge and incentive to avoid.

    • Viliam says:

      mistake pedagogy vs conflict pedagogy

    • Reasoner says:

      To play devil’s advocate for a minute, I think one could argue that the process may be more important than the motivation. Like, suppose you act like an arrogant jackass and try to prove me wrong by asking the toughest questions you can think of, even though you don’t have any curiosity… but in the course of doing so, you reveal an important inconsistency in my understanding.

      I dunno about you, but I would like to think that if I was in that situation, I’d be able to swallow my pride and accept it as a gift. As Deng Xiaoping said, “No matter whether the cat is black or white, if it catches mice, it is a good cat.”

      To play devil’s advocate again for what I just said, suppose you have no curiousity about this topic, and there’s no particular benefit to me enlightening you (because you have no use for this knowledge and will quickly forget it), and also I feel pretty confident that your conceptual jabs won’t ultimately reveal any weakness in my understanding. In that case it seems reasonable for me to ignore you, because sharing my knowledge with you isn’t a good use of my time, and it’s also tiring me out.

      So I think there’s a good argument for being humble and trying to communicate curiosity along with asking the toughest questions you can think of, because (a) it communicates to your interlocutor that you truly want to be enlightened, (b) it offers social status to your interlocutor in exchange for the info they share with you, (c) it makes the conversation more satisfying for your interlocutur.

      However, it’s possible that this mode of operation doesn’t come naturally to people, because it involves simultaneously being “arrogant” enough to think that it’s worthwhile for them to come to your own understanding instead of just taking the other person’s word for it, while simultaneously (play-acting?) as “humble” in order to get someone to feed them information. My current guess is that it’s still the way to go if you can master it.

      Also, all bets are off in an online context, where it’s been alleged that the best way to get the right answer is to confidently post the wrong answer 😉 (This may be less true in modern internet culture though)

    • ProbablyMatt says:

      In an online context, or an offline context with an audience, I think it is still beneficial to answer the questions even if the asker is not motivated by curiosity but indeed trying to find a gotcha. The reason is that others who are witnessing the exchange can deepen their understanding of whatever you’re trying to teach, and also learn that you are a person who will take questions seriously and try to address them. This should increase their confidence that you are knowledgeable and making a good faith attempt at educating them.

    • AllAmericanBreakfast says:

      I’m taking a second tour through undergrad, prepping for a career change. I’ve also been a teacher for the last 10 years. This gives me some personal experience with what Scott’s writing here.

      Only a few people in my classes ask questions. When they do, their questions are on point. Often, others share their confusion, which I know because students, including myself, discuss those interactions after class. We also discuss their discomfort with asking questions. We express fearing taking up too much class time, or seeming arrogant. We also express frustration when a teacher fails to answer a question appropriately, and deep appreciation when the teacher spends an hour working step by step through a challenging concept during office hours. I’m at a community college, but taking STEM classes, so there is some selection for motivated students.

      There are students in class who are shopping on Amazon or scrolling on Facebook on their laptops during lecture, which you can see if you sit in the back of class. These students don’t ask questions. Students who are checked out have much more interesting things to do than mock the teacher. They might also look like idiots if they tried. At least in college, their peers are paying for the class, and since mocking is an attempt to claim status, they’re risking losing that status by revealing their ignorance. So Scott’s analysis makes perfect sense to me in a classroom setting.

      The dynamic is different on the internet, in mandatory schooling, and in lower-achieving classrooms. My students are children, and I teach music lessons. They frequently do things to disrupt the lesson. But it’s *never* mocking me by asking relevant questions.

      The closest thing they do is play stupid. They pretend to have forgotten things, and make me explain it all over again to them. They are not very sophisticated at this, so they’ll pretend to have forgotten so much that I know they’re lying. I deal with this by pretending to be sympathetic, and telling them that we’ll just have to go back to playing “little kid songs” again to help them relearn. And then asking “if they’re sure” they don’t remember, and cueing them a little with the intellectual tools they have to get them started.

      Mostly, kids slack off by squirming around, telling pointless stories, zoning out, or needing trips to the bathroom/to get a drink of water/to put on a sweater.

      But plenty of my students engage in Socratic grilling, and this is very easy to tell apart. They’re in obvious distress, there’s a confrontational atmosphere, the questions are about the subject matter at hand, they’re persistent, and they consistently display signs of being engaged with learning.

      The trouble with the internet is that you don’t really have a chance to get to know the questioner. Status isn’t transmitted the same way. In real life, status among classmates evolves gradually, from your comportment in the classroom throughout the quarter or year. Knowing when to stay silent and give space to others is as much a part of it as making your voice heard.

      On the internet, status is directly associated with your comments – your voice. You can only acquire it by posting, and whatever status you accrue can’t be translated to your other comments, or to your online persona as a whole. The only exception is if you become internet-famous enough that your account has name recognition. And even if that happens, there’s none of the other subtle pleasures of the physical camaraderie of the classroom. Sitting with your classmates can have a raw, animal pleasure of sharing space together engaged in the same task. Spending time together outside class is another outlet for status-exchange.

      So I think your theory about there being a large percentage of trolls engaged in faux-Socratic grilling is most likely to be true on the internet. But I’m really not sure about my estimate for how much of internet grilling is trolling and how much is a genuine attempt at learning. My gut says that 1% learning vs. 99% trolling is way off, especially on this blog. There’s too many people who actually care about the subject matter, and too little in the way of reward for trolling (no comment karma or like button) for this to be a good environment for status-seeking through trolling.

      But I think also that there’s a risk that blogs themselves are seen as trolling attempts. Scott could be seen as a massive internet troll by an unsympathetic reader. If he were, he’d be getting quite a bit of status for his success.

      I think the real challenge isn’t distinguishing trolls from Socratic grillers. I’m a mistake theorist at heart.

      So I think it’s that we get so focused on attainable small goals that we outright have to set aside related goals. The communist is so focused on the immediate goal of developing their communist analysis of the world economic system, that they have to set aside the alternate goal of considering arguments for capitalism. Given infinite time and energy, they would do so, but by their lights, developing the communist analysis is so much more pressing than considering arguments for capitalism that it would be a mistake to try.

      Anybody who comes in trying to get them to do so is being disruptive. So they get designated a troll and smacked down. They’re asking good questions in the wrong classroom. Smacking them down is the internet’s way of informing them of that fact.

      My guess is that 99% of even the most trollish internet questions are good questions asked in the wrong classroom, though this does not extend to trollish remarks more generally.

      • Simon_Jester says:

        The dynamic is different on the internet, in mandatory schooling, and in lower-achieving classrooms. My students are children, and I teach music lessons. They frequently do things to disrupt the lesson. But it’s *never* mocking me by asking relevant questions.

        Yeah. The big problem is the cases where someone mocks you by asking irrelevant questions (e.g. questions that are about questioning the teacher’s authority to keep order, not about the subject material).

        I don’t have a response to that issue that I feel good about, becuase I want to be able to justify why I’m telling them to do things, but ultimately the answer is “because mechanically it’s my job to teach 20+ people at once, and I can’t do that if we don’t observe rules of order which, yes, include you doing what I just told you to do, Timmy.”

    • albatross11 says:

      Bret Weinstein has a great line about this: Bad faith changes everything.

      If I can assume you’re asking questions in good faith, then I can answer them literally and clearly as possible. That’s what I do when talking with my kids, for example–if I explain Schelling points or viral replication or kin-selection with them, and they ask this kind of question, I’m thrilled because it means I’m getting through. But if I’m in an environment where someone’s likely to be trying to play monkey status games with me, or to obscure an issue via dark arts/rhetorical tricks in order to win an argument or derail a conversation, then it’s hard to know how to answer those same questions sensibly.

  2. Kaitian says:

    I think you underestimate how often questions like that are actually meant as attacks. For many question askers, your answer is completely irrelevant. “Why didn’t you do the dishes when I clearly asked you to?” “Where is the cake that I bought yesterday?” are not attempts to find something out, they’re criticism.

    Even the skeptic / rational sphere actively advocates something like: when someone states a position that seems bad, ask them questions about it until they realize it’s stupid. You’re not actually expecting the anti vaxxer / religious fundamentalist / psychic medium to convince you of their beliefs, you just want to call them out.

    And then there’s the added complication that you can’t really answer a question to 100% satisfaction. You can always go up or down a level of detail, and continue arguing. How do we know germs are real? Isn’t it privileged to talk about hygiene when many people don’t have access to clean water? Etc. There’s no obvious line between legitimate questions and people “just asking questions” to annoy or silence you because they don’t like your argument.

    Tl,dr: considering how questions are actually used in conversation, you can’t prevent people from interpreting your questions as hostile.

    • 2dipsynock says:

      You’re not actually expecting the anti vaxxer / religious fundamentalist / psychic medium to convince you of their beliefs

      I think you’re misunderstanding Scott here. It’s not just about being convinced of their beliefs, it’s also about understanding them well enough to even know what it s they believe. Scott has obviously put effort into understanding Communism – that’s not just to “dunk on the communists” at the end.

      Your response feels like it’s very skewed to one off anonymous interactions. In person (like with a teacher-student relationship) it’s pretty obvious by the second or third iteration when the student either thanks you at the end or starts shouting conspiracy theories.

      you can’t prevent people from interpreting your questions as hostile.

      Obviously no question phrasing will prevent this 100% of the time, that’s an unreasonable goal. That has no bearing on there obviously being better and worse ways to phrase things.

      • Kaitian says:

        Your response feels like it’s very skewed to one off anonymous interactions. In person (like with a teacher-student relationship) it’s pretty obvious by the second or third iteration when the student either thanks you at the end or starts shouting conspiracy theories.

        Scott was talking about his online interactions, so that’s where I started too. But even in person, it’s easy to read a socially awkward person as hostile. It’s just less defensible, because hostile askers in person are often pretty identifiable by things like their tone of voice or their overall relationship to you.

        I do agree that there are better and worse ways to phrase things, I just think that (in online / impersonal discussions) the chance of a question being interpreted as hostile depends much more on the expectations of the person being questioned than on any characteristic of the question itself.

      • Garrett says:

        > Scott has obviously put effort into understanding Communism

        I remember an interaction some time ago where the commentariat had made a serious attempt to understand what Marx was saying in one of his pieces, and were being guided by one of our Marxist commenters. The frustration on both sides was palpable despite it being done in good faith. Inferential distance is as good of a term as any.

    • Aapje says:

      @Kaitian

      “Why didn’t you do the dishes when I clearly asked you to?” “Where is the cake that I bought yesterday?” are not attempts to find something out, they’re criticism.

      These questions are actually at least in part an attempt to find out why you didn’t do the dishes or what happened to the cake. That the person who asks the question may already suspect that the answer is an unacceptable one (to them), doesn’t change the fact that the question itself doesn’t presume an answer.

      This is different from an allegation that does presume the answer, like: “You refused to do the dishes because you don’t care about me!” or “You ate my cake!”

      You’re not actually expecting the anti vaxxer / religious fundamentalist / psychic medium to convince you of their beliefs, you just want to call them out.

      Rebutting a person challenges both them (to make their case) and yourself (to make your case). An inability to do the latter (well enough) can lead to a change of opinion (which doesn’t have to mean adopting the beliefs of the other person, but can be a middle position or replacing a claim with a more nuanced/defensible one).

      • Kaitian says:

        These questions are actually at least in part an attempt to find out why you didn’t do the dishes or what happened to the cake.

        That’s fair, but most of the time the person already knows the answer and just wants to add some additional humiliation by making you say it. In person, it’s easy to tell by their tone of voice which case applies. Maybe you know nicer people than I do, so you’re reading the questions in a different tone.

        Rebutting a person challenges both them (to make their case) and yourself (to make your case).

        The way the technique was presented to me was explicitly intended to change the other person’s mind, understanding their position was not part of the goal. Fundies DESTROYED with FACTS and LOGIC, that kind of thing.

      • carvenvisage says:

        These questions are actually at least in part an attempt to find out why you didn’t do the dishes or what happened to the cake.

        I don’t think so for the first question: the superflous “clearly” in the first question is a clear marker that the question is not intended in an exploratory manner.

        But “where is the cake I bought yesterday” could be a genuine question altogether. The only indication otherwise is that “where is” wasn’t abbreviated. (which is probably a slightly stronger indication that it sounds like)

    • Edward Scizorhands says:

      If someone is asking a question on the Internet, where there are lots of people around, and you have the choice of dunking on them for being difficult, or answering their question, or not saying anything at all, you should not say anything at all instead of dunking on them.

      It’s fine to not want to bother answering their question. But let someone else do it. Or say “that’s a good question, but I don’t have time for it right now. Can someone else explain it?”

      • albatross11 says:

        +1

        Also, if you want to communicate, both with the person you’re talking to and bystanders/readers, then give up on snark and sarcasm and outgroup-bashing and ingroup signaling and everything else, and just focus on stating your positions as clearly as possible.

    • Spookykou says:

      I think the most central example of a question as an attack is “Why?” on League of Legends.

    • unreliabletags says:

      The Programmer’s Stone claims that the “mapping” kind of thinking, of assimilating new information into a connected graph of understanding as opposed to a stimulus-response rule (“packing”), is rare. Mappers don’t realize this, and go around committing typical-mind errors all the time.

      He uses this to explain the conflicts between knowledge workers and the business people who want to do Taylorism on knowledge work. For example, the difference between a checklist as a reminder to consider certain topics (mapper view) vs. a set of required steps to take, or to punish someone for not taking (packer view).

      It may explain this too. If your interlocutor has never had the experience of trying to fit a knowledge packet into his graph and encountering tension, he will not have empathy for what you’re trying to do, and will of course interpret the question as hostile.

    • J Mann says:

      Even the skeptic / rational sphere actively advocates something like: when someone states a position that seems bad, ask them questions about it until they realize it’s stupid. You’re not actually expecting the anti vaxxer / religious fundamentalist / psychic medium to convince you of their beliefs, you just want to call them out.

      I would say that a good rationalist would recognize that there’s a possibility that the anti-vaxxer might be correct and might convince the rationalist, but that she estimates the probability of the first statement to be very low.

  3. Riverless Uniform says:

    Sometimes, the teachers don’t have the explanations. And it looks bad in front of other people.
    Even though it is very often true, the teacher can not think, much less say publicly, “This course is only about putting you in a room, making you more docile/disciplined/able-to-just-take-it, making you learn stuff that builds common knowledge around what we culturally value, and what we culturally value superficially looks like” rather than “Actually understand stuff”.
    We want most people to know that alchemy is pseudoscience, while chemistry is, and what those two look like so that they can discriminate. Not for most people to grok the scientific method so that they can make the judgement by themselves.

    It’s the same thing with ideologies and religions. It’s not only about classrooms.
    Of course there are people trying to understand stuff in religions, ideologies and classrooms. But as pointed in the OP, most people just don’t expect stuff to make sense.

    Even rationalists and nerds quite selectively expect stuff to make sense. I have seen countless times an overly literal person trying to smash down woo, religious practices, ideological beliefs or obviously-wrong-things-TM in an attempt to publicly or privately gotcha, because it feels good.
    Especially when there are useful locally internally consistent things that they leave off the table, to focus on the gotchas.
    Trying your identity to understanding more real things than others has some drawbacks.

    I do not believe it’s worthwhile for Scott to distance himself from those people, because making the distinction is too costly (both for Scott and the people making the distinction).
    I like the link as a disclaimer, and think it would work better if when he expects a post to go viral, he signals who the target audience is (people trying to figure out stuff or something).

    • Cerastes says:

      Sometimes, the teachers don’t have the explanations. And it looks bad in front of other people.

      Not IME. In my classes, occasionally students will ask something I don’t know, and I’ll say as much. I’m in biology, though, where the sheer breadth of pure facts precludes anyone knowing everything or even being able to derive it from prior knowledge (we joke that we’re the first science to have a single, universal answer to everything, and that answer is “Well, it’s complicated…”).

      • xenon says:

        Yeah, this was my experience in school as well. I went to a very good district, but generally when I asked something a teacher didn’t know, they would either say something like “I’m not sure, I’ll see if I can find out” or they would challenge me to look for the information on my own.

        My mother is a teacher as well, and she says that modeling how to find information is at least as important as actually passing information along.

        • Loriot says:

          I’ve heard stories that lead me to believe the median teacher is considerably worse than that (as in probably less knowledgeable about the subject matter than the average poster here was at high school age).

          • Viliam says:

            You are right. People who had good schools can easily underestimate how bad it can be at an average school. The good schools are good exactly because, among other things, they “steal” good teachers from other schools. Schools have a zero-sum competition for good teachers; you can make a school great, but you can’t make the entire school system great that way.

            I believe this is inevitable, unless the entire educational system is dramatically changed. Let’s look at numbers. If we assume that people live 70 or 80 years on average, and they spend 12 years at elementary or middle schools, that means they spend about 1/6 of their life at school. (I only include elementary and middle schools here because in the developed countries pretty much everyone has them, and because they are almost the same. Universities are a bit different, so I ignore them here.)

            In the current system of education, you need about 1 teacher per 10 or 15 pupils. That means that 1-1.5% of the entire population needs to become teachers at elementary or middle schools. Suppose the entire workforce is, dunno, half of the population, or maybe a bit more, it means you need 2-3% of your workforce to be teachers.

            We could have an utopistic debate about what skills and traits we expect from a good teacher. But forget that! If you need to hire 2-3% of the entire workforce, you can’t really set your standards high. It’s even worse, because not everyone who could be a good teacher, also wants to be a teacher — a person who is good at learning and understanding things can also become a scientist, a doctor, a programmer, an entrepreneur, etc.

            I don’t really see a way to provide good education to everyone, unless we change that “1 teacher per 10 or 15 pupils” parts. Yeah, one option would be to only provide education to a fraction of population; they we would need fewer teachers, and we could be more picky about them. (I don’t really want this — I am just saying this is an explanation why education in the past used to be better in certain aspects: if you only have a handful of students and handful of teachers, you don’t have to hire the incompetent ones.) In absence of genetic engineering that would make the average person less stupid and more curious, the only remaining option is to change the teacher:student ration in some way. I see two things that could be done, they are not mutually exclusive:

            1) Separate a role of teacher from “the one who maintains discipline in the classroom”. Because one important thing that limits the size of the class is teacher’s ability to maintain discipline. Explaining stuff to hundred people is not inherently more difficult than explaining stuff to ten people. (It becomes more difficult if they ask questions, because now you have more questions. But a typical class is not like this.) What is really difficult is maintaining a discipline in a group containing more disruptive students. I could imagine a class having 40 students instead of 20, with two adults: one who would explain things, and one who would maintain order.

            2) Use more television and computers. The teachers typically say the same stuff over and over again in various classrooms each year. Let them make a video instead! Now if you say “but what if a student wants to ask a question”, you have a good point, but I could imagine a lesson consisting of e.g. 30 minutes with video and 15 minutes of live debate with the teacher. But during those 30 minutes of video, the same teacher could be in another classroom, having their 15 minutes of live debate there.

            I know that people have an objection against all these proposals. But I don’t really see any way how to magically convert a sufficiently large fraction of the population into competent teachers. (And even if we had a way to magically produce experts, there are things besides education where we could use them.)

          • merisiel says:

            Heck, I went to a fancy private elementary school and I had bad experiences with this. There was one teacher there in particular who seemed to feel somehow threatened by students asking questions he hadn’t prepared for (especially in science).

          • I don’t really see a way to provide good education to everyone, unless we change that “1 teacher per 10 or 15 pupils” parts.

            You suggest videos, but an older approach is books. You can have a model in which imparting information is done entirely by books and the teacher is there only for interaction. That seems to be mostly the Oxford model for higher education.

            Another possibility is to have students teaching students. That not only increases the number of teachers, it also helps educate those teachers, because teaching something is often a good way of learning it.

            Heck, I went to a fancy private elementary school and I had bad experiences with this.

            I went to what may have been the best private school in the state of Illinois, run by the University of Chicago. The physics teacher, confronted with a proof that a claim of the driving ed text was false, responded by insisting that the claim was true, refusing to answer the proof, and insisting that because he was the teacher what he said was right.

            I have long regretted that I didn’t encounter him after I got a PhD in physics, to point that by his own credentialist standards he was now wrong.

          • Simon_Jester says:

            @Viliam

            We could have an utopistic debate about what skills and traits we expect from a good teacher. But forget that! If you need to hire 2-3% of the entire workforce, you can’t really set your standards high…

            This is one of the most insightful comments I have ever seen about education, from someone who is not (as far as I know) themselves an educator.

            We talk about how teachers are not good enough, but the simple brute-force reality is that when we hire teachers, we don’t have the luxury of doing more than select for “has an education in the 60th percentile or higher” and “has taken some relevant courses and training or has relevant experience in teaching” and “can fulfill the legal requirements to establish to our satisfaction that they aren’t a sociopath or a child molestor.”

            Because otherwise, we run out of warm bodies to put at chalkboards, the class sizes balloon, and you get problems that more than cancel out any improvement in standards for teachers.

            I don’t really see a way to provide good education to everyone, unless we change that “1 teacher per 10 or 15 pupils” parts. Yeah, one option would be to only provide education to a fraction of population; they we would need fewer teachers, and we could be more picky about them…

            This actually makes less difference than you’d think, because if you only educate a fraction of the population, you will find, realistically, that you’re not educating the smartest fraction of the population.

            Go back to the 19th century and 19th century portrayals of, say, British boarding school education. You will find plenty of teachers who snap at students for asking questions, plenty of teachers who are downright sadistic in ways that we’d screen right the hell out of a modern school system ASAP, and plenty of students who aren’t particularly motivated to learn and just hire a ‘crammer’ to help them pass their mandatory graduation-requirement exams.

            There is nothing new under the sun, at least not in the dynamics of how pedagogy actually works.

            If you only provide education beyond the sixth-grade level (or less) to the upper and upper-middle classes, or any other corresponding elite, you are forced to hire teachers from that elite and from that elite alone. You MAY get on average a slightly higher quality of teacher, but you won’t get radical improvements, because a lot of the people who would have made good schoolteachers simply cannot check the relevant educational checkboxes and so sigh and become a laborer in the monocle factory or whatever.

            1) Separate a role of teacher from “the one who maintains discipline in the classroom”. Because one important thing that limits the size of the class is teacher’s ability to maintain discipline. Explaining stuff to hundred people is not inherently more difficult than explaining stuff to ten people. (It becomes more difficult if they ask questions, because now you have more questions. But a typical class is not like this.) What is really difficult is maintaining a discipline in a group containing more disruptive students. I could imagine a class having 40 students instead of 20, with two adults: one who would explain things, and one who would maintain order.

            This is an extremely good idea, especially if combined with mechanisms for removing chronically disruptive students from the classroom environment, ascertaining what their problem is, and figuring out how to deal with them. Modern schools try to do this, but we simply don’t (at least in the US anywhere I’m familiar with) have the manpower to do it to the extent required. Especially when parents are resistant to admitting their child’s behavior is a problem, which in my experience is typically an upper/middle-class disease.

            2) Use more television and computers. The teachers typically say the same stuff over and over again in various classrooms each year. Let them make a video instead! Now if you say “but what if a student wants to ask a question”, you have a good point, but I could imagine a lesson consisting of e.g. 30 minutes with video and 15 minutes of live debate with the teacher. But during those 30 minutes of video, the same teacher could be in another classroom, having their 15 minutes of live debate there.

            This works, but only in a classroom setting that is highly ordered and monitored. You need a reasonable assurance that the children aren’t slacking off, and most mechanisms I can think of for doing that either circle back to (1) “hire more discipline monitors” or become Orwellian techno-horror in pretty short order.

          • Viliam says:

            @DavidFriedman:

            You suggest videos, but an older approach is books. You can have a model in which imparting information is done entirely by books and the teacher is there only for interaction. That seems to be mostly the Oxford model for higher education.

            Yes, the basic idea is the same. Videos could be more accessible for younger pupils who don’t read fast enough; and would allow better visualisations.

            Yet another option is to make wiki pages. I imagine that a teacher could gradually transition from classical teaching to wiki — all they need is to update the wiki a bit after each lesson.

            @Simon_Jester:

            This is one of the most insightful comments I have ever seen about education, from someone who is not (as far as I know) themselves an educator.

            Thank you, but I actually taught computer science at high school for about five years. Then I returned to software development.

            It is interesting to see the contrast between the two professions. As a software developer, I could hypothetically create a software once, and then millions of people could use it. As a teacher, if I want to teach 60 kids something, I need to make four groups by 15 kids, and tell it to each group separately.

            If you only provide education beyond the sixth-grade level (or less) to the upper and upper-middle classes, or any other corresponding elite, you are forced to hire teachers from that elite and from that elite alone.

            Good point.

    • gleamingecho says:

      It seems to me that the epistemic humility required of the type of thinker/expert many teachers are expected to be in the teaching aspect of their jobs can sometimes be at odds with the authoritative forcefulness that is also required of many teachers in the “teach kids how to submit to authority” role they often have.

      This tension is probably most present in, e.g. a high-school social studies teacher, where her teaching knowledge to teaching submission roles are roughly equivalent on the spectrum, whereas a kindergarten teacher would be much more focused on the submission aspect and a college lecturer would be much more focused on the knowledge aspect.

  4. jesduff says:

    How much evidence is there that somebody who starts off curious, with an innate desire for the world to make sense, could actually become less so over time through having a bad teacher who conflates curiosity with arrogance? Isn’t the desire for the world to make sense in many ways down to an intolerance for uncertainty, which is a personality trait that’s hard to change?

    I’m trying to imagine how this plays out in practice. The kinds of teachers who routinely shun curiosity and give bad explanations are typically disliked by students, and so if you’re a student being shunned by your teacher for your curiosity, maybe you’ll take it with a grain of salt and piggyback on existing negative sentiment towards that teacher by talking shit behind their back to restore your self-confidence? Even if you took the teacher’s scorn to heart, come recess time aren’t you going to be associating with friends who are likeminded and far more likely to entertain a curious mind?

    It seems like a whole lot of things need to go wrong pretty early in life to fundamentally make you feel like curiosity is a bad thing, and even if from early childhood everybody told you curiosity was bad, I don’t think it’s possible to make somebody less curious on a fundamental level.

    • 2dipsynock says:

      This seems like learned helplessness. There’s research on whether that applies to curiosity, but I haven’t read it. Check Google scholar.

    • Purplehermann says:

      Curiousity being a bad thing would be extreme.

      Not being excited by curiousity, being unwilling to bother looking for answered seems more likely

    • ksdale says:

      While reading this post, I kept thinking of parents, rather than teachers. Small children can ask a ton of questions, and it can get suuuuper annoying, but I’m still shocked by the number of parents who actually get annoyed by their children asking questions in good faith.

      My wife and I try really hard to make our kids understand that it is *always* ok to ask questions and that it’s important to understand things, but I think a huge number of children are constantly implicitly taught that questions are annoying.

      • eremetic says:

        I hope you’re not giving them a lecture on how great asking questions is every time they ask a question!

    • MawBTS says:

      The kinds of teachers who routinely shun curiosity and give bad explanations are typically disliked by students

      My experience is that the teachers liked by students are the ones who don’t hand out detentions, let class out on time, tell their students what material will be on the test, etc.

    • Chris Phoenix says:

      I would have to write a long essay to support in detail what I’m about to assert: Your post misses some important parts of childhood psychology and social dynamics, and as a result, you presuppose the non-existence of mechanisms of harm which in fact routinely do great harm to children’s intellect.

      For example, a large fraction of the people I talk to (in a well-educated demographic) learned that they were “bad at math” via having a math teacher who did not alleviate their confusion.

      For example, being overtly silenced by a teacher costs a child several kinds of status and safety; the child may learn (correctly!) from even one experience that it’s a bad idea to try again.

      For example, I’m told by an expert that if you don’t test girls for giftedness by 4th grade, it’s too late – they’ve learned to hide it in order to fit in, and they’ll hide it even on the test.

      I’m not sure you’re right about the desire for the world making sense being related to intolerance for uncertainty. A person who enjoys jigsaw puzzles may be highly rewarded by seeing the pieces fit into place, but also has to sit in front of hundreds or thousands of pieces out of place for hours at a time. For me, software engineering is much like this – many things don’t fit into place, and that’s OK because they will eventually (even if that place is the “less important than I thought” bucket).

      But even if you’re right, humans can learn to tolerate almost anything. (This may be another piece of psychology that you’re not taking into account.) They may be less happy, they may avoid thinking about large parts of reality, but if they find themselves in a social context where it’s better to ignore problems, they will ignore those problems with a vengeance. Abusive relationships, cults, American politics, unhappy marriages that last decades, the Milgram shock experiments… the evidence is overwhelming that most people can and will do this.

      And in the context of learning and thinking, our ability to tolerate things that aren’t right can quickly become a vicious cycle, leading to the exact kind of shutdown you’re arguing against.

      I could keep typing for hours, but if you aren’t yet rethinking your position, I doubt more words will make much difference.

      • whereamigoing says:

        For example, I’m told by an expert that if you don’t test girls for giftedness by 4th grade, it’s too late – they’ve learned to hide it in order to fit in, and they’ll hide it even on the test.

        Do you have a citation for this?

  5. Rachael says:

    This is a great and important concept, but I think it’s badly named, and I hope that if it catches on and becomes part of the rationalist memeplex it does so with a better name.
    IIRC, Socratic questioning is when you are right and the person you’re questioning is wrong, and rather than just telling them they’re wrong, you ask them questions to probe their ideas and expose the flaws in them, and make them come to the conclusion that they’re wrong by themselves.
    And, as you say, “Socratic grilling” sounds like a harsher version of this process.
    So it seems like a bad idea to use it to describe almost the opposite process, where you believe you’re probably wrong and you ask the expert a series of probing questions to try to identify where your own misunderstanding is.
    If, in the future, A tries to justify their persistent questioning of B by describing it as “Socratic grilling”, that’s not going to make B any less defensive or less annoyed, or reduce B’s perception of A as arrogant and hostile; it will probably have the opposite effect.

    • 2dipsynock says:

      So it seems like a bad idea to use it to describe almost the opposite process

      This is a really great point.

      “Platonic Questioning” because instead of Socrates asking his pupil questions it’s his pupil questioning him?

      “Questioning Socrates” is probably just a clearer version of that idea, with a bonus connotation along the lines of the “I know I am but a mere student” disclaimers.

    • John Schilling says:

      you ask them questions to probe their ideas and expose the flaws in them, and make them come to the conclusion that they’re wrong by themselves.

      Meh, it’s easier to come to the conclusion that you should be put to death for corrupting the youth of Athens, er, the Internet.

      You’re right that there’s a disconnect between what Scott is advocating, and what Socrates was famous for. In Scott’s version, the person asking the questions is genuinely interested in learning from the answers. The actual Socrates version has the questioner as a smug know-it-all who only cares about the answers as a means for judging the answerer, and the utilitarian justification for that lasts no longer than the patience of the person being questioned by the smug know-it-all.

      • Dragor says:

        Yeah, death penalty is bad and free speech good and all, but I could see where Socrates’ condemnors were coming from….

        • John Schilling says:

          If I recall correctly the jury voted about 280 to 220 in favor of conviction, for a crime where the median sentence was ten years’ exile. Then Socrates argued for a lighter slap-on-the-wrist sentence in the penalty phase. The jury voted about 360 to 140 in favor of having him put to death.

          So, prolonged exposure to the Socratic method can cause 36% of people who don’t think the speaker is guilty of committing any crime, that he nonetheless ought to be executed.

          Friends don’t let friends do the Socratic method.

          • Simon_Jester says:

            The Socratic method has two key components:

            1) Use of questioning to undermine the certainty of the subject. This includes various techniques and sub-criteria. It can be very annoying on the receiving end, but a reasonable degree of mental self-discipline and the love of wisdom can counteract the annoyance.

            2) Use of deliberately outlandish or extreme claims as rhetorical flourishes, or to illustrate and support a philosphical abstraction by promoting its uttermost extremes in practical life.

            It is this second part that got Socrates sentenced to death, in my opinion; the first part is relatively safer and is more in line with what we remember as the “Socratic method.”

          • John Schilling says:

            It can be very annoying on the receiving end, but a reasonable degree of mental self-discipline and the love of wisdom can counteract the annoyance.

            Self-discipline and love of wisdom on the part of the Socratic “teacher”, or the student? Because the student usually has the option of finding a different teacher, and occasionally of penalizing the needlessly annoying one.

        • Viliam says:

          It’s the classic “torture vs dust specks” scenario. Killing Socrates destroys fewer QALYs than letting him annoy the public.

          For the same reason, spammers should get death penalty.

          • John Schilling says:

            It’s the classic failure mode of consequentialism, where doing the math right is really hard but people are sometimes really really motivated to reason “It’s OK for me to kill this annoying git because something something trolleys mumble dust specks”.

      • Protagoras says:

        I think Socrates is portrayed as trying to figure things out for himself and at least some of the time as trying to learn from the people he’s questioning (though admittedly most often eventually using them as sounding boards in his own explorations once he’s determined they don’t really know anything useful).

    • Rand says:

      IIRC, Socratic questioning is when you are right and the person you’re questioning is wrong, and rather than just telling them they’re wrong, you ask them questions to probe their ideas and expose the flaws in them, and make them come to the conclusion that they’re wrong by themselves.

      There’s a Wikipedia page on Socratic questioning, and it doesn’t sound like that at all:

      Socratic questioning (or Socratic maieutics)[1] was named after Socrates. He utilized an educational method that focused on discovering answers by asking questions from his students. According to Plato, one of his students, Socrates believed that “the disciplined practice of thoughtful questioning enables the scholar/student to examine ideas and be able to determine the validity of those ideas” [2]. Plato described this rigorous method of teaching to explain that the teacher assumes an ignorant mindset in order to compel the student to assume the highest level of knowledge [3]. Thus, a student has the ability to acknowledge contradictions, recreate inaccurate or unfinished ideas and critically determine necessary thought.

      Do you have examples of “Socratic questioning” being used in the sense you’re describing?

      • Spookykou says:

        you ask them questions to probe their ideas and expose the flaws in them, and make them come to the conclusion that they’re wrong by themselves.

        asking questions…a student has the ability to acknowledge contradictions, recreate inaccurate or unfinished ideas and critically determine necessary thought.

        I would guess the difference in tone is in response to a disagreement with the framing as a teacher student interaction, my memory(which could be totally wrong it’s been a while) was that a lot of the dialogues are not actually with ‘students’ except in a sort of, all of humanity are students to Socrates kind of way.

  6. Markus Ramikin says:

    Single datapoint, but I remember as a former very smart kid (now a slightly smart middle aged man), I used to experience both: a desire to make things make sense (and an endless annoyance at school’s refusal to help me in that), AND a smug satisfaction from proving people dumber than they thought they were, including teachers. In retrospect, there’s no way a teacher could have known which mode I was in at any given time. Probably, most of the time both motives were at work.

    Kids are human, and many humans will be assholes at your expense when they can get away with it. Even the ones who are genuinely trying to think and understand.

    • Purplehermann says:

      Teachers who actually knew what they were talking about and made sn effort to answer my questions as though they were earnest (regardless of whether they were) pretty quickly shifted me into earnest only with them.

      The smug satisfaction with a teacher would only happen when I didn’t respect them, answering earnestly earned my respect.
      Was it similar for you?

    • Doesntliketocomment says:

      Piggybacking on this, answering these questions in a classroom can sometimes be unhelpful. As a bright young child I would frequently ask questions in class that would further my understanding but were at best useless to my classmates, at worst downright unhelpful by introducing confusion in the basic concept they were trying to understand. My teachers generally handled this very well, however, by telling me I could talk with them later about it, or giving me an indication where I could find more information.

    • theodidactus says:

      I’m no longer a practicing catholic, but my confirmation saint is Vitus, a roman comedian, which I chose because I liked the method of his conversion. One legend at least said that he converted to Christianity midway through a comedic performance of christian rites…he was in the process of making fun of christians when he realized they had the right idea all along.

      I’ll cop to being one of those snotty students that tried to show how smart he was by asking probing little questions…but in the process I learned a lot, sometimes by realizing my questions weren’t really that clever.

      When I taught, I always tried to give students lots of room to ask questions and/or show off. There’s an alternative theory that your job is supposed to be to teach them life lessons about not being a know-it-all or whatever, but frankly, I think it’s more productive just to use it as another entry into the subject being taught. I’m saying this in the light of once again being a student there’s two ways that stuff sticks in your head the strongest:
      1) when you were right and everyone else was wrong and you could prove it
      2) when you were wrong and everyone else was right and you had to admit it.

      EDIT: That link attributes the story to st. Genesius rather than St. Vitus,but the story is what matters.

    • Protagoras says:

      Plato’s dialogues give the impression that Socrates had both motives at work as well.

    • Wolpertinger says:

      Twenty years later and I am still that guy, but the reward function has become more muddied and complicated. Now there always is the nagging doubt whether I am overdoing it, i.e. just being a smartass and asking pointless questions or whether that impulse to ask something is a genuine concern. It is hard to reflect on that while in the middle of a discussion.
      On the other hand working on actual projects allows me to see the actual outcome weeks or months later and earn the occasional “told you so” reward.

      What makes me most uncertain in my own behavior is the difference in frequency of asked questions. Some people ask none or maybe one at the end of the presentation/discussion of the day while there’s just few people who interject frequently.

  7. slate blar says:

    I actually love disclaimers, because I’m terrible at detecting implicit ones(even in my own thinking! if I’m not careful I’ll accidentally feel overconfident about something).

    The darkside of Socratic grilling is that it can be done in bad faith to just troll, waste time or disrupt a discussion. To be fair in an actual classroom teachers should have a sense of who is being a trouble maker and who is earnest. On the flipside the troll being told off can discourage earnest questioners.

    How do you actually tell the difference?

    • 2dipsynock says:

      In a classroom I think the student’s earnestness vs troublemaking isn’t that important compared to whether the answer is instructive. On the internet I think people frequently overestimate how much of the value of their conversation is to the person it’s directly to vs. the passive audience that sees both sides, so the same consideration applies. You’re never convincing the troll, but you can sometimes convince an audience.

      I spend a fair amount of time politely yelling at people on reddit & feel like when I suspect someone might be trolling my main considerations are whether writing a response will help me clarify my own thoughts (especially if my argument relies on a shortcut like “everyone knows this”) and whether I think rebutting the troll will keep a passerby from believing something stupid.

      The intent of the possible troll doesn’t end up mattering that much in a lot of situations.

    • Error says:

      I don’t know how to tell the difference between good and bad faith questioning, but I have some heuristics….

      * If there’s an audience, it’s more likely in bad faith (point scoring).
      * If it’s a one-off interaction, it’s more likely in bad faith (random trolling).
      * If there’s no audience and it’s someone you know, it’s more likely in good faith — or at least it’s easier to judge based on prior interactions. TFT works.
      * If you’re an Actual Teacher and the other party is paying you for your time, it’s almost certainly in good faith.
      * …actually that last probably generalizes to any costly signal of desire to learn.

      If you want to teach people who want to learn, and don’t care about scoring points yourself…then charge for your time. Students will pay for answers. Trolls and status-slappers won’t — not just because of the money, but because the act of paying someone is on some level an acknowledgement of their status and defeats the point.

      (NB: charge the *student*, not a third party. If a third party pays, that is only evidence that the third party wants the student to learn, not that the student themselves wants to learn.)

      …on further reflection, I wonder if a practice of charging nominal amounts for answers to questions would raise the signal-to-noise ratio of a lot of internet environments.

  8. alext says:

    Hi and thanks for this very interesting blog.

    To me, it sounds like the student isn’t very accepting of the new information that the teacher is providing.

    The implicit assumption in a teacher-student relationship is that the teacher *has something important to teach* that the student *is currently ignorant of*. Otherwise, why participate at all?

    Allowing this, the student needs to accept a certain amount of non-congruence with his current world view. Instead of immediately questioning each and every perceived piece of nonsense, the logical reaction is to store the new “strangeness” alongside the rest of the new data until it will hopefully, eventually, make sense.

    The teacher’s response could very legitimately be “You don’t know enough yet for this to make sense. In fact, right now, I’m providing you with that data. So shut up and let me finish. Then think about it. Then, ask away.”

    • Purplehermann says:

      That’s a legitimate response sometimes, but still requires the teacher to assume the student is earnest, and really is confused

      • alext says:

        but still requires the teacher to assume the student is earnest

        I think it does not require that assumption. The “shut up and let me finish” answer ought to satisfy an honest learner (more data is coming, wait for it, it’ll click eventually) as well as silence a troll. The value in silencing a troll is that it allows the teacher to continue teaching any other honest students. This might be why it’s not an uncommon reaction.

        • Randy M says:

          “shut up and let me finish”

          “Hold on, I’m getting to that” is very common to hear in a classroom. But if the teacher says “Now that we’ve established how to balance equations, we can move on to stoichiometry” and you’re still wondering what all those letters are, it’s a good clue to ask questions.

          (That wasn’t a great example, since it’s not showing the student asking an insightful question. )

          • merisiel says:

            Even non-insightful questions can lead to this roadblock. My private lesson teacher one summer at music camp was convinced that I was resistant to correction — if I didn’t understand an instruction he was giving me and asked questions about how it worked, he accused me of “strawmanning him” and pretending not to understand so that I wouldn’t have to change what I was doing.

            (Boy, was I glad my long-term teachers in my hometown weren’t like that.)

    • Aapje says:

      @alext

      The implicit assumption in a teacher-student relationship is that the teacher *has something important to teach* that the student *is currently ignorant of*.

      Yes, but this assumption can be (partially) wrong. It is useful for the student to recognize whether the teacher actually has something important to teach and if so, what parts of his teachings are important. Plenty of people with expertise in A, but not expertise in B, still discuss B as if they are experts.

      Otherwise, why participate at all?

      Students are often not free to choose their teachers or choose their teachers based on recommendations by those who can’t necessarily evaluate the teacher for their competence.

      In fact, the catch-22 of teaching is that the only person who can fully evaluate a teacher is one who doesn’t need to be taught. The more someone needs to be taught, the more susceptible they are to bad teachers.

      So shut up and let me finish.

      This is sensible if understanding requires set of concepts/facts to be understood, but not when a questionable or not understood claim is made that is treated axiomatically for the rest of the teaching.

      • alext says:

        this assumption can be (partially) wrong

        When the student stops believing it, he stops being a student (ie someone who wishes to learn).

        the catch-22 of teaching is that the only person who can fully evaluate a teacher is one who doesn’t need to be taught

        I’d argue that complete competence is not required to judge someone else’s ability. I’m not a plumber, but I can identify a bad plumber by the amount my toilet is leaking. One makes a value judgment and hopes it’s not too far off. Wishing for perfect information has many names. I’ll stick with unrealistic.

        not when a questionable or not understood claim is made that is treated axiomatically for the rest of the teaching

        All new claims are questionable and not understood, some remain that way right until the end of the lesson. Some require years and years of hard work to make sense of. Still, contradicting them would fully negate any learning whatsoever. As an example, the basic assumption of relativity (light moves at the same speed in all frames of reference) is thoroughly outlandish. Yet one must accept it or remain ignorant. It all ends up making sense, but only much later.

        • Robert Beckman says:

          Your plumber example doesn’t work because you’re only able to discern a useless plumber from a bad plumber.

          Example: your toilet is leaking under the foundation, the plumber replaced the plumbing with a) ground contact rated PVC or b) non-ground contract rated PVC. In both scenarios your immediate results will indicate that she’s a good plumber: your toilet isn’t leaking, but over the course of a few years the difference will arise, because the competent plumber knew to install the correct material.

          Real example: my mother had a new roof installed, with new insulation. As part of this she paid more for a radiant barrier, with the contractor installing OSB roof decking with pre-applied aluminum paint (aside: this is a very good and appropriate product), in addition to the new fiberglass insulation. However, the contractor installed the roof decking with the shiny side up, in direct contact with the roof membrane and asphalt roof shingles, explaining to my mother that this way the heat would be reflected back out. After the installation she observed that temperatures were indeed cooler under the roof than before, and concluded that this was a competent contractor. For those who haven’t figured it out yet: the contractor was incompetent. A radiant barrier requires an air gap between the reflective/emissive layer and the adjacent layer, because a radiant barrier only stops radiative heat and does nothing for conductive heat. By place the reflective side in direct contact with the roofing membrane the contractor ensured that there would be no radiative transfer through the material, and all energy transfer would be conductive.

          So how does a layman identify when the expert is not in fact an expert?

    • merisiel says:

      The implicit assumption in a teacher-student relationship is that the teacher *has something important to teach* that the student *is currently ignorant of*. Otherwise, why participate at all?

      Allowing this, the student needs to accept a certain amount of non-congruence with his current world view. Instead of immediately questioning each and every perceived piece of nonsense, the logical reaction is to store the new “strangeness” alongside the rest of the new data until it will hopefully, eventually, make sense.

      This seems more like religion than science to me, tbh.

      Also, I think that some back and forth is often needed: the ability to say “Well, I understand that bit, but here’s the thing I’m having trouble reconciling with these other facts” can help quite a lot. Though, of course, it’s not always feasible (if you have a lecture hall with 100 students, they can’t all be doing this).

  9. Purplehermann says:

    The flashbacks… assuming arrogance/status grabbing with kids is evil

    • sarth says:

      I think I have similar flashbacks.

      But to be fair, it’s not always inaccurate. Kids can have complex motivations too, and not even be aware of their own layers.

      • Skeptical Wolf says:

        But to be fair, it’s not always inaccurate.

        This is true, but misses an important point. The teacher in these interactions is clearly communicating to the student and every member of the audience “this is what happens when you ask questions”. Assuming arrogance or status gaming from a child is destructive and evil, even when it’s correct.

        • Murphy says:

          There’s a common theme with student teachers.

          “I’m gonna be different! I’m gonna be that cool teacher!”

          And sooner or later the students break them.

          Ever been in the room when a young teacher breaks?

          And for every student who’s legit just trying to figure things out… there’s dozens or hundreds who just get a kick out of playing status games and screwing with the authority of the adults supervising them.

          Society needs that rare student who legit is trying to make the world make sense.

          But society needs them because they’re so rare.

          The vast majority of their fellows are not seeing the world that way and are mostly concentrating on the status games.

          Unless their students are somehow screened for the kind of kids who act like that out of genuine good faith or is teaching at Uni level…. any teacher who tries to ignore that will find normal teaching almost impossible… until they break or quit.

          • Skeptical Wolf says:

            And sooner or later the students break them.

            In other fields, we have a word for this “your job is to interact with a group of people in pursuit of a goal and you can no longer effectively do so” phenomena. We call it “failure”. But when we encounter it, we regroup, recover, learn from the experience, and do better next time.

            If someone fails and stays failed, then they’re in the wrong job.

            But society needs them because they’re so rare.

            In my experience, they outnumber status-questioners at all levels of education. But they’re usually accustomed to being slapped down whenever they ask a question, so it takes effort to connect with them and draw them out.

            Ever been in the room when a young teacher breaks?

            Yes. And I was there to help them come back afterwards. No one (except perhaps the teachers themselves) expects perfection. But if you can’t act like the people you’re there to teach are there to learn, your students aren’t the problem.

          • Simon_Jester says:

            @Skeptical Wolf

            In other fields, we have a word for this “your job is to interact with a group of people in pursuit of a goal and you can no longer effectively do so” phenomena. We call it “failure”. But when we encounter it, we regroup, recover, learn from the experience, and do better next time.

            If someone fails and stays failed, then they’re in the wrong job.

            If the vast majority of the people who try to do a job in good faith on the terms you outline for doing the job “fail” by your description of “failure,” there comes a point at which you must revisit your defined parameters for proper performance of the job.

            Either:

            1) You need to devise a mechanism by which teachers can easily eject bad-faith actors from the room (along with genuinely disorderly and disruptive students who cause the question/answer format to break down), or…

            2) You need to reduce class sizes so far that every question any student might plausibly have about the content can be addressed in full, combined with ejection of disruptive and disorderly students as per (1), or…

            3) You need to recognize that there are times when the logistical realities of teaching students in batches of 20+ force the teacher to say “not now” or “ask me later” or “pipe down and let me finish explaining, THEN think for five minutes, THEN ask any questions that survive the above process.”

  10. ExplodingCabbage says:

    A similar but sort of opposite kind of offence-taking can happen when the apparent error you’re questioning turns out to actually be an error. Then instead of people being offended at you challenging the authority of your betters, they’re offended at you bullying your inferiors. Why couldn’t you just plainly state that the claim was in error, they ask you? Why did you have to disingenuously pretend to be confused? Did you just want to mock someone who didn’t know something that you knew?

    All of which makes me hate everyone who demands disclaimers one way or the other. When something seems wrong to me, often it’s equally possible either that I’m missing something or that it really is wrong, and that’s fine. It should be completely socially acceptable in most contexts to simply object to something that seems false, and sometimes turn out to be right, and sometimes get a persuasive rebuttal fired back at you.

  11. Florent says:

    My first reaction when reading this wasn’t that the student was arrogant, but rather that germs look a lot like an invisible dragon in my garage.
    If a teacher went this far into the conversation without showing me a microscope, I would seriously doubt their theory.
    Conclusion: it’s much easier to develop a germ theory if you’re dealing with cholera than with the common cold.

    • Rachael says:

      Yes, that was my first impression too. I thought it was leading into an essay about how explanations that look like made-up special pleading are sometimes correct, and applying this to some political or cultural issue.

    • Simon_Jester says:

      This is a large part of why the germ theory of disease took 200 years to invent and popularize after the invention of the microscope. For much of that time, the primitive state of biology made “serious epidemic diseases are caused by self-replicating micro-organisms that hitch rides in or on living organisms, surfaces, and liquid media” little or no more believable than “serious epidemic diseases are caused by toxic vapors rising off of conspicuously smelly things such as feces,” to pick one example of a competing hypothesis that wrestled hard with germ theory during the 19th century.

      Today, we more or less ask people to accept this on authority, because it is REALLY IMPORTANT for preservation of public health that everyone be on the same page with respect ot public sanitation.

  12. Murphy says:

    I get the impression that the culture in research and at universities, postgrad level is waaaaay better suited to this kind of person.

    But I think that’s for a few reasons.

    1: The kind of people who believe in mysticism and bullshit tend to have been filtered out. Either because they failed their tests for writing “gods will” when asked what can cause genetic mutations or because they couldn’t stand being around heretics any longer or they deconverted. This allows far more default assumptions of good-faith.

    2: high time investment to get there, someone who just want to stick it to the materialists and show them that magical crystals fields are the real source of all knowledge isn’t going to keep going long enough to end up there.

    Indeed the kind of students who ask those kinds of questions tend to be the most promising. We had one a little while back who probably every few hours came out with something like “but why can’t we just do [blah] to figure out [foo] by doing [buzz]”

    Each time you could see some grins light up around the room and often the answer was along the lines of “absolutely but [missing piece of the puzzle]” or “sure but [unresolvable ethical problem for a step]” or “definitely but if you can figure out [buzz] then you’ll be in line for a nobel prize”

    When she finished up we pretty much all agreed that we’d enthusiastically accept any application from her to work there in future.

    • zzzzort says:

      I would say that for someone doing original research, a certain amount of arrogance is required. To think that someone is going to find a fundamental fact about the universe that know one in history has noticed before is the height of hubris, but it’s also the requirement for a PhD. Maintaining that arrogance is especially hard in a university environment that is so full of smart, accomplished, knowledgeable people.

    • sharper13 says:

      Would it surprise you to learn that a third of scientists in the country believe explicitly in God? Over half, if you add as well in those who believe in a universal spirit or higher power?

      Reading your comment, I’d suspect that you don’t have a lot of experience in discussions with rational religious people, but you probably know more of them than you think, you just don’t catch that they’re religious because of the environment you know them in.

      But agreed, looking at the data, the university/academic culture is significantly more atheist than the rest of the country (with the exception of Jews, who are a higher percentage of the scientific community than they are of the general population, for what are probably obvious reasons).

      • Murphy says:

        There’s plenty of people who are culturally religious, baptised and go to church at Christmas or who had their bar mitzvah but don’t bother much with temple.

        There’s plenty who like the community around a religious group.

        There’s some who are philosophically religious, who believe in the abstract idea of a creator god.

        The woman who sits in the next office is a fairly observant muslim who’s got into fights with the local imam’s over their views on women.

        But there are very very few of the kind of people who would answer “gods will” when asked what can cause genetic mutations, the kind of people who believe in an interventionist, busybody deity that alters things on the fly to mess with us because that doesn’t mesh well with science and the idea of there being any point to scientific investigation.

        • sharper13 says:

          So your position is that believing in God (at least enough to respond to a survey about it that way) isn’t “mysticism and bullshit”, but there is some large subset of people who answer “gods will” about everything, which then disqualifies them for science?

          Maybe consider instead that you have a strawman idea of what most people who believe in God are like and how they think. I know a pretty decent number of believers and none of them match your description. It could be that I’m just super lucky in my acquaintances, but maybe also you have biased sample or are making some unwarranted assumptions?

          • Murphy says:

            more than 40% of americans believe that the world is 6000 years old.

            A similar fraction (41%) believe The Rapture is imminent.

            A similar fraction either entirely reject the idea that evolution is a thing that happens or believe that any observed change is the result of His Noodley Appendage His Divine Touch changing creatures according to His will.

            That’s a lot of people.

            They are less numerous in europe but they’re still there. Derren Brown, a stage magician talked about how he used to be a member of an evangelical church growing up…. then he started gigging, going stage magic card tricks and his own church turning up at one of his first stage shows to pray in tongues loudly at the back and they wouldn’t believe that it was just slight of hand, they literally believed in magic and that he was communing with the devil.

            They share little in common with the people I grew up going to church with.

            But they exist, they are numerous and their world-view is utterly insane.

            It’s weird but a lot of people seem somehow utterly unaware of how many of their fellows are completely off the deep end.

          • Hyperfocus says:

            @Murphy

            You may be confusing belief as attire with genuine belief. Back when I was religious, I had the “wonderful” experience of believing both my religious “facts” and my science facts. For instance, if you had asked me directly “did Native Americans originally come to America from Jerusalem in 600BC?” I would have felt compelled to say yes. But if you went by my behavior in an anthropology class, as long as you didn’t confront my religion directly, you would assume that I believed that Native Americans came to America across the Bering Strait 20,000 years ago, because I did.

            Belief as attire is fundamentally performative, and so when you directly ask questions that challenge belief like “is The Rapture imminent?” you will get performative answers. Ask instead if those people invest in their 401ks. If they really believed The Rapture was coming this year or next, they wouldn’t bother saving for a tomorrow that will never come.

  13. Well... says:

    Typo: “an kid”

    • Well... says:

      Found another one: in the paragraph that begins “Tolerating this” there’s a period or semicolon missing between “one or two iterations isn’t always enough” and “I’ve had times when I need five or ten”.

      (Just trying to be helpful.)

  14. viVI_IViv says:

    . And interpreting an attempt to explore dissonance as a status grab that needs to be knocked down is absolutely fatal for that skill. Instead, you need to think of it as Socratic grilling – like Socratic questioning, but a little harsher and more confrontational in order to get to the point more quickly.

    And how did this work out for Socrates? Just asking, Scott-sensei, no sarcasm intended… 🙂

  15. Gabriel Conroy says:

    Slightly off-topic, but Socrates was a jerk. He focused on tripping people up and putting words into their mouths. A student, or someone who sincerely wants to understand, is a different matter. But Socrates himself is not someone I like. (I’m referring to the Socrates of the (admittedly few) dialogues by Plato I’ve read. I have no strong opinion about the historical Socrates, if he even existed.)

    By the way, have you read “Interrogating Socrates” by Beversluis (I forget the author’s first name). Beversluis looks at Plato’s early dialogues and points out a lot of points where the character Socrates falls short of the, erm, Socratic ideal. As Beversluis presents it, the arguments are often shallow or fallacious, and Socrates often fails to grant his interlocutors the minimal respect they deserve. He does say that Plato’s later dialogues don’t suffer from those shortcomings, at least not as much.

    I’m not sure how to judge that book. I read only about 1/3 of it, plus the conclusion. But it was a good read.

  16. JohnBuridan says:

    Yesterday I was discussing supply and demand as it relates to the second industrial revolution. The purpose was to segway from there to concentrated urban poverty, from concentrated urban poverty to anarchism, from anarchism to The Man Who Was Thursday.

    A fearless student, known for Socratic grilling raised his hand:

    Student: isn’t it more expensive to build the factory and machines than to just have the workers make more of the product or higher more workers?
    Teacher: Well, sometimes that is true. But when I say ‘more efficient,’ I mean anything which lowers supply curve. It doesn’t have to be bringing in machines. Frequently machines are not brought in until well after they are available.
    Student: But why would you ever want the machine? The machine is costly and needs repairs and has all sorts of other costs? Why not just use humans?
    Teacher: It depends on the context of what is being produced. Many times it eventually becomes obvious that machine can do significantly better than many individuals whom you would higher AND the machine quickly pays for itself. If I imagine paperclip production, I think a machine can make 10s of thousands of paperclips a day, but a person can only make a few hundred.
    Student: But we are talking about iron works and clothing factories. When does it become worth it then?
    Teacher: You seem concerned that it wasn’t worth it to move society into industrialization, is that right? Let’s take the fact that it did happen as evidence that some people thought it worth it. If you want to know more about why it was worth it, we can talk after class to see if we can make sense of it.

    The student shrugged a “you have not succeeded in making sense to me” shrug. The worst part of this situation was how distracted and annoyed the students were by his questioning. The entire time a student or two wanted to jump in and “help” make the teacher’s point. Sidenote: students cannot be trusted to do this. If this student wasn’t basically immune to social pressure, he would have shut himself up after half a question. The other students are basically the internet. Some of whom want everyone else to accept what I have to say as Gospel because I am a teacher. How dare another student waste their time by questioning me.

    • JohnBuridan says:

      We also wound up discussing Tribe and Against the Grain as modern examples of Rousseau’s noble savage and the idea that modern society has serious costs worth considering.

    • Murphy says:

      I’d honestly probably offer the student a mini assignment worth a few marks asking them look into the history of how many paperclips or springs a skilled human can make by hand per hour and how many hours per year they can work at top speed and how much such a person would cost to hire per hour.

      compared to the cost of a few of these:

      https://www.alibaba.com/product-detail/Automatic-CNC-wire-paper-clip-making_60633909098.html?spm=a2700.7724857.normalList.1.61cd35e5XAMDeV&s=p

      It feels weird that I can look up how much it would cost me to buy a major piece of factory equipment on my credit card to be shipped to my home….

    • Deiseach says:

      I like the mental image of you rolling off on your Segway as you segue into the next part of the talk, followed by your students who are being forced to stroll quickly and hold up a conversation at the same time 🙂

      Student: isn’t it more expensive to build the factory and machines than to just have the workers make more of the product or higher more workers?

      You’d think, but once the initial high cost is over, the machines make up for it over time as they are faster and more productive than human workers, and you don’t have the expenses of paying wages to extra workers to be hired on if you want to up production. This was the entire fight of the Luddites versus factory owners who wanted to automate their mills, and if your smart-arse student wants some flavour of the times, recommend them to read Charlotte Bronte’s Shirley which, amongst other things, gives the rationale from the mill-owner’s side against the loom-smashers (if I could handle it when I was nine, they can force themselves to read some 19th century prose):

      When he came to Yorkshire, he—whose ancestors had owned warehouses in this seaport, and factories in that inland town, had possessed their town-house and their country-seat—saw no way open to him but to rent a cloth-mill in an out-of-the-way nook of an out-of-the-way district; to take a cottage adjoining it for his residence, and to add to his possessions, as pasture for his horse, and space for his cloth-tenters, a few acres of the steep, rugged land that lined the hollow through which his mill-stream brawled. All this he held at a somewhat high rent (for these war times were hard, and everything was dear) of the trustees of the Fieldhead estate, then the property of a minor.

      At the time this history commences, Robert Moore had lived but two years in the district, during which period he had at least proved himself possessed of the quality of activity. The dingy cottage was converted into a neat, tasteful residence. Of part of the rough land he had made garden-ground, which he cultivated with singular, even with Flemish, exactness and care. As to the mill, which was an old structure, and fitted up with old machinery, now become inefficient and out of date, he had from the first evinced the strongest contempt for all its arrangements and appointments. His aim had been to effect a radical reform, which he had executed as fast as his very limited capital would allow; and the narrowness of that capital, and consequent check on his progress, was a restraint which galled his spirit sorely. Moore ever wanted to push on. “Forward” was the device stamped upon his soul; but poverty curbed him. Sometimes (figuratively) he foamed at the mouth when the reins were drawn very tight.

      In this state of feeling, it is not to be expected that he would deliberate much as to whether his advance was or was not prejudicial to others. Not being a native, nor for any length of time a resident of the neighbourhood, he did not sufficiently care when the new inventions threw the old workpeople out of employ. He never asked himself where those to whom he no longer paid weekly wages found daily bread; and in this negligence he only resembled thousands besides, on whom the starving poor of Yorkshire seemed to have a closer claim.

      • JohnBuridan says:

        It’s part of our neo-deweyan educational approach. Teachers talk slow but segway fast.

        The Bronte quote is useful! Thanks.

  17. MartMart says:

    I like to preceed my round of question with “this is such an obvious objection that I’m sure I am not the first to think of it, so can you tell me how X has been addressed?”

  18. I think actual teachers get this right more often than not. What I’m concerned about is the self-appointed Internet anti-autodidact brigade. I occasionally get these people as commenters on my blog, but they’re more common in Reddit and places like that.

    There’s actually a tendency popularly associated with Redditors, which is summed up by the phrase “I f*cking love science!”, and it involves a love of pop scientism and scientific authority over the actual process of discovery. I think it stems from people’s interactions with the Reddit atheist community.

    In fairness, Reddit at least has a million subreddits, so some are based on the idea of “no bad questions”. The “Explain like I’m 5” subreddit is even based on the idea of explaining things right from the very basics in the way the youngest possible child student would learn.

  19. NovaByblos says:

    [perhaps overly on-topic]
    re: germ reservoir. Can I add a new question at the end? I’m not five, but I’m also not a medical expert. Can someone actually explain to me how this works, and why the kid’s plan, with some added “exterminate some local populations of raccoons, etc…” wouldn’t work? I get the general concept, and I understand that warm weather suppresses viruses, making it seasonal, I guess I just don’t understand the economics/tradeoffs of “it seems like in the summer, the total number of bats/raccoons/etc… that contain the virus would be pretty low, and come fall when it starts to spread, the number of initially infected flu victims is low, and sure, maybe it costs a lot (billions?) to test/track them down/quarantine humans/exterminate animals, but the flu costs billions of dollars in damages each year from deaths, illness, lost work hours, so how is it not worth it?”

    I don’t mind if you call me an idiot, just make sure the insults are paired with a good explanation. 🙂

    • NoRandomWalk says:

      Because quarantining everyone is impossible. Set aside the trillions of dollars in lost productivity, direct costs, a huge number of people just won’t agree to do it and then you need to have a bunch of draconian state hired thugs go about enforcing it by interacting with these ‘defectors’. And now you have people interacting. So you don’t have a quarantine.

      Also, it’s not just racoons, it’s needing to disinfect all the surfaces, its…literally stop eating or producing food, I think? because all species can give us diseases, not just the ones we don’t eat; and already it’s barely worth it to get flu vaccines. I don’t think we need to do anything drastic.

    • Edward Scizorhands says:

      XKCD had a “why don’t we just isolate everyone” but I think it was in his book, not online, so I can’t link it.

      I also got interrupted in the middle of reading that chapter so I would like to know how it ended!

      • Randy M says:

        Ha, I was just searching for that too! From memory, Randal concluded it would work but isn’t practical do to space concerns or something.
        But I imagine eventually some of the benign bacteria we live with will mutate into a pathogenic form even if we killed off the presently virulent ones by mass quarantine. Viruses I’m not sure about.

  20. Roebuck says:

    It would be great if we could invent a very short disclaimer to signal whether we’re feeling confused rather than confrontational. By short I mean requiring less effort than inserting a hyperlink to this post.

    It could be something short that you can put at the beginning of the sentence, like “Let me understand – […]”.
    As in: “Let me understand – if X was really true, wouldn’t that imply Y?”.
    However, if many people start using it, I’m afraid some people will try to use it sarcastically and then it will become just a euphemism for ‘gotcha’.

    How about something more self-deprecating: “I’m confused – […]”. But I guess this can still be turned into a sarcastic jibe.

    How about something more cooperative, something that uses “we” / “us”.
    For example, “What if we ask – […]”, “Let’s consider – […]”. I think these sound better and the latter one is my current favourite.

    EDIT: Let’s throw in more ideas.
    “I’m trying to understand. […]”
    “Entertain this – […]”
    “So, […]”
    “Hmm, ” <- This one is maybe trying too much to convey defensive non-verbal communication in written form.

    • sclmlw says:

      I think the whole problem is the classic challenge of translating non-verbal communication into written form. In a face-to-face conversation you can pick up on lots of cues that tell you the other person is sincere. Even over the phone you can pick up cues like this, which is why it’s so helpful to just call someone to resolve problems as opposed to trading emails.

      If the problem is that written language strips all non-verbal cues away, it won’t be possible for Scott to return those cues with a simple link or even with short phrases like you suggest. Literally trying to replace a whole type of sub-lingual communication with a quick link or phrase won’t work. Because it isn’t the same thing.

      I’m reminded of a book I read about spoken language, where the author talked about useful words that have no denotation. They are useless in print, but they signal information when used in conversation. For example, phrases like, “um”, “I mean”, “Seriously, though” don’t translate into meaningful communication in text form. But if you’re talking to someone and they say, “I mean…” then they pause to take a breath, you’ll assume they’re going to continue the thread of what they were just saying and would like you to not interrupt them.

      A nod of the head, a raised eyebrow, or a slight squinting of the eyes communicate a lot about how the speaker’s communication is being received by the listener. A diverse array of nonverbal cues goes the other direction as well.

      You can’t replicate that in the written word. Text is a different medium. It’s better to stick to the strengths of text as a communication form, and compensate for its weaknesses, as opposed to trying to recapitulate the elements of conversation into written form. One of those compensations may be that you can’t do Socratic grilling in written form. Unless you’ve previously engaged with that person verbally.

    • Don P. says:

      In its context, this is the function of that “explain like I’m 5” thing: it announces “I’m not being a dick, I honestly don’t understand this”.

    • carvenvisage says:

      The trick is to furrow your brows and look confused.

      (but make sure you look kind of stupid rather than perplexed by balderdash)

    • Jiro says:

      It would be great if we could invent a very short disclaimer to signal whether we’re feeling confused rather than confrontational.

      How do you prevent confrontational people from intentionally using the disclaimer?

  21. Nick says:

    A lot of people have already objected in the same way I was prepared to: it is hard sometimes, especially at first, to distinguish Socratic grillers (and I agree btw with @Rachael’s point that this is a bad name) from people asking in bad faith or who are stupid or lacking other intellectual virtues. Others have pointed out the teacher can be put in a really awkward spot here (and some think that’s a good thing, and others think that’s a bad thing, and it seems clear to me that It Depends). Still others say other students make this more difficult.

    I’m going to try to unify these critiques: Socratic grilling requires certain conditions to work well. Some are environmental, while others are virtues held by students and teachers.

    This is easiest to see by looking at some ways Socratic grilling fails:
    1. Your example, but the student ends by saying he’s unconvinced. He goes away thinking, “Man, that teacher was making it up as she went along. Pfft! Germ theory is stupid. I’m gonna go rub dirt on my wounds.” The student is the one missing something here. He might have seemed like a Socratic griller, but in reality he had no understanding that the teacher has something to teach him in the first place. He’s missing the virtue of intellectual humility.
    2. Your example, but the student is acting in bad faith. This is hard to tell, but eventually the questions get distracting enough that the teacher has to put off the discussion—as many teachers would. The student however shows no interest in following up: he’s just wasting class time or trying to confuse other students. He’s acting in bad faith. Notice, by the way, that it’s impossible to really tell from one case the student is acting in bad faith. You may notice a trend, which is the sort of thing being a teacher teaching a particular class enables, but in other places, good luck.
    3. Your example, but the teacher runs into a question she can’t answer. She hits him with a ruler for mouthing off or tells him some things just can’t be explained or tells him it’s all in the book. Obviously, the teacher is at fault here; she’s abusing her authority. Another way to think about this is that she’s abusing the trust her students place in her to be able to ask questions forthrightly.
    4. Your example, but the teacher runs into a question she can’t answer. The student pounces, declaring victory, and announcing confidently that the teacher has nothing more to teach him. This is another failure of intellectual humility.
    5. We’re two random commenters, and you’re the expert trying to persuade me utilitarianism is true. I continue to repeat a stupid argument according to which Omelas has proved utilitarianism wrong. No matter how many times you explain the same thing, I don’t get it, and more commenters come along to say you’re wrong for more stupid reasons. What went wrong here? The conditions are just awful. You should have known going in you’re not going to persuade your interlocutor; the best you can hope for is persuading some lurkers. Your mistake was going on the Internet.
    6. Same as above, but the conversation is going more productively. Still, I end every new objection I’ve just come up with twelve seconds ago with things like, “This is why utilitarianism is false” and “Therefore your view conflicts with simple and well known facts about human nature”. You begin to get seriously annoyed and remind me that I’ve abandoned my earlier objection at least three times already. What went wrong here? The Socratic grilling was working, but you still felt like you were talking to a brick wall. You don’t know me, and we don’t have that relationship of trust that a student and teacher have, so I come across as kind of an ass. The mistake was mine; I should have used a disclaimer.

    We can see in the last case that sometimes disclaimers are worth it. I don’t think it’s possible to make every place a place where Socratic grilling works, because it’s ultimately a matter of etiquette, and in our culture etiquette is not uniform among all people and all places.

    • Randy M says:

      I think the best case for it is one-on-one without an audience, where no one has to lose status for admitting to be wrong or ignorant. In which case, if they get you, well, so what, you’ve learned about their character and lost nothing, and if they are genuine, your patience may reward you both with better understanding.

    • sclmlw says:

      Wouldn’t the simple solution be for the teacher to ask questions of the student to understand better where they’re coming from? That way you avoid a lot of guesswork and confusion about whether someone is asking in good faith. You also ensure you’re addressing the underlying assumptions the student has, instead of expecting them to meet you at the level of understanding you’re coming from.

      Socratic grilling implies a conversation. If there’s a back and forth, it’s unclear to me why the relationship has to be unidirectional in terms of information. Presumably the teacher has information to give to the student, but in order to communicate that information they need information from the student. So just ask.

      • soreff says:

        Socratic grilling implies a conversation. If there’s a back and forth, it’s unclear to me why the relationship has to be unidirectional in terms of information. Presumably the teacher has information to give to the student, but in order to communicate that information they need information from the student. So just ask.

        Yes, agreed. One other thing that can help is for the student to volunteer similar information. One frequently helpful way to do this is for the student to paraphrase what the teacher said and ask if this indeed matches what the teacher intended or not.

  22. alwhite says:

    What happens when this is happening on both sides simultaneously?

  23. Barry says:

    The materialist is like a person that denies the narrative contained within a book. Imagine looking at a book and saying there’s nothing here but cardboard and paper and little black marks on the pages. It seems the reality of “meaning” disproves materialism.
    The soul is the narrative contained in the book of the body. The narrative gives purpose and meaning to the existence of the material or physical parts of a book.

    • snahgle says:

      What do you mean by the “reality of meaning”? Do you mean that it feels real, and is therefore real in the same way that atoms are real? I think that meaning is something that pretty clearly happens on the level of thought (e.g. ‘in software’) rather than on the level of cells (e.g. ‘in hardware’). Your web browser is real, but materially it exists in a heavily-encoded way as a combination of electrons and processing patterns. That doesn’t mean that the existence of operating systems disproves materiality.

      • Barry says:

        Yes, the existence of operating systems disproves materialism. Operating systems are ideas made real through microchips and electricity and the interpretive power of the human mind. Living things are ideas brought to life in the flesh.

        • Murphy says:

          Yes, the existence of operating systems disproves materialism.

          Really… no.

          You’re describing entropy and encoded information.

          They exit purely in the physical realm. The information encoded in atoms and electrons.

    • mendax says:

      Whether a book has meaning beyond its materials is not always an easy question to answer.
      Experts are uncertain if the Voynich Manuscript is meaningful, or not.

      Are the books in the Library of Babel meaningful? Are the ones that contain language that one understands more meaningful than the others?

      • Barry says:

        Analogy is an immaterial concept found in most human languages.
        Sometimes a hoax is the immaterial purpose of a book.

    • Faza (TCM) says:

      It seems the reality of “meaning” disproves materialism.

      And what is “meaning”, exactly?

      I think we can broadly agree that “meaning” is – at the very least – a property of words in living, natural languages, as in: “words have meaning”, yes?

      This being the case, what is the meaning of the word “pies”?

      • Barry says:

        It might be better to frame it this way, words convey meaning depending on context. Context is another way to recognize the reality of meaning. Like when we laugh at the jokes in a silent film.

        • Faza (TCM) says:

          I’m not sure I am following you.

          Could you please unpack “context is another way to recognize the reality of meaning” a bit?

          • Barry says:

            If I go into a bakery the word “pie” will most likely signify something different than if I were in a math class.
            I’m not sure if that helps.

            My sons really enjoy the children’s books by Aaron Becker they are wordless story books with just pictures but my sons are able to tell what’s going on in the story just from looking at the pictures. The material images are able to communicate a meaningful narrative.

    • TheAncientGeeksTAG says:

      If the extra element that extracts meaning is immaterial soul stuff, then it will you have a robust statement of I’m materialism.. but the materialist can always respond that what extracts meaning is computational processes in the brain.

  24. snahgle says:

    This reminds me a bit of a critical response process invented by Liz Lerman (a choreographer). Details here. In short, there are four steps: (1) Saying what you noticed about the thing, (2) the artist asking the audience questions, (3) the audience asking the artist neutral questions, and (4) the audience offering opinions that the artist can decide whether they want to hear.

    This was intended for improving art criticism rather than generalized learning, but one of the key points is that stating your opinion about the thing happens last, because it is almost always less useful than describing what you noticed about it and asking neutral-valence questions about the intent. For teaching/learning, figuring out how to ask, receive, and answer questions in a neutral way (i.e. one that doesn’t imply an opinion) is helpful in both directions for avoiding devolution into less useful conversations.

    • Glenn says:

      This is neat, thanks. Repeating my comment on this from elsewhere: I think the kind of structured discourse presented therein is an underutilized tool, probably because it takes a lot of trust and effort from the participants.

  25. Alex M says:

    Here’s the problem – a lot of times when people question you, they really are trying to do “gotchas.” And the best solution for that is to recognize their hostile questioning as the attack that it is and to simply respond with equal hostility.

    For example, when I point out that sociology has almost no ability to replicate (while memetics does), or that economics has almost no ability to predict (while game theory does), and that sociology and economics are not real sciences but just pseudosciences using complex math to masquerade as real sciences, a lot of economists and sociologists are going to lose a lot of status, money, and respect if the public starts listening to me. Naturally these people on the losing end of the paradigm shift are going to hate the idea that I am right, so they will attack me with hostile lines of questioning. They will deliberately mischaracterize what I am saying, pretend not to understand me, or spread false rumors, all so that they can avoid having scientific consensus converge on an unpleasant truth: that they are peddlers of bullshit who deserve to go out of business for misleading society. Of course they will try to do this in a sneaky way that disguises their attacks as “just asking questions!” because they are pretending that they operate under mistake theory and that they are just trying to advance science, even though their true motive is entirely selfish. These people couldn’t give a damn about science, they simply are afraid that any new scientific paradigm will expose them as the frauds that they are. There is no amount of justification that I could ever give most sociologists or economists that would convince them that their entire field of study is delusional nonsense, because as Upton Sinclair said: “It is difficult to get a man to understand something, when his salary depends upon his not understanding it.”

    This is why paradigms that operate under mistake theory advance science slowly, while paradigms that operate under conflict theory advance science fast. If I stop trying to convince sociologists and economists that memetics and game theory are superior to their own science, and instead weaponize these disciplines (for example, by helping politicians that are hostile to traditional sociology and economics achieve political victories through the use of memetics and game theory) then the public has no choice but to accept these facts because any political group that refuses to accept the new paradigm will rapidly go extinct.

    To give you an analogy, imagine you encountered a primitive tribe and were trying to convince them that their ignorant superstitions were false and should be updated in favor of science. It would be very difficult to convince their shamans and chieftains that your paradigm was real while theirs was fake, since part of their tribal authority stems from the legitimacy of their paradigm. But if you pull out a machine gun and say “Let me demonstrate how physics and chemistry works” and put a few holes in whatever primitive medicine-man is most hostile to you, the tribe will update their beliefs with a real quickness because not only have you demonstrated the effectiveness of your science in a very tangible and unquestionable way, but you also have fewer critics who are willing to speak against you openly.

    In the rationalist community, most questions are Socratic grilling, and you can usually give the benefit of the doubt to those people and use mistake theory in your dealings with them. But outside the rationalist community, people tend to be less concerned about truth and more concerned about their own status, position, and power, which is why it makes more sense to apply conflict theory in those situations. The reason rationalism has had such a hard time spreading even though it is objectively correct is because people like you are wasting time trying to convince the metaphorical village shaman that they are wrong when you should simply be pulling out the metaphorical machine gun. Giving the benefit of the doubt to people who have a strong vested interest in refusing to update their beliefs is a waste of time.

    • Viliam says:

      paradigms that operate under mistake theory advance science slowly, while paradigms that operate under conflict theory advance science fast

      I suppose we can agree that the paradigms that operate under conflict theory definitely advance something fast. The only remaining question is whether that something is actual science.

      by helping politicians that are hostile to traditional sociology and economics achieve political victories through the use of memetics and game theory

      …you sometimes get this.

      • Alex M says:

        …you sometimes get this.

        I believe I already said “any political group that refuses to accept the new paradigm will rapidly go extinct” but thank you for reiterating my point so effectively.

        • Simon_Jester says:

          What was said was not a reiteration of your point, it was an addendum:

          “Any political group that refuses to accept the new paradigm, and any group favored for destruction by the new paradigm whether it opposed the new paradigm or not, will rapidly go extinct, and this very easily becomes a huge net utilitarian negative.

          There is also the practical problem that, well, “war does not prove who is right, only who is left.” Most of us here would not readily accept the premise that the victory of the Reds in the Russian Civil War of 1917-20 proves the superiority of Communist ideas over the ideas of other factions in the war. I imagine that most of us would say the war didn’t prove anything about whose ideas were better or more accurate. We’d probably say it proved a great deal about which faction was most ruthless in organizing and in eliminating its enemies.

    • mendax says:

      To give you an analogy, imagine you encountered a primitive tribe and were trying to convince them that their ignorant superstitions were false and should be updated in favor of science. It would be very difficult to convince their shamans and chieftains that your paradigm was real while theirs was fake, since part of their tribal authority stems from the legitimacy of their paradigm. But if you pull out a machine gun and say “Let me demonstrate how physics and chemistry works” and put a few holes in whatever primitive medicine-man is most hostile to you, the tribe will update their beliefs with a real quickness because not only have you demonstrated the effectiveness of your science in a very tangible and unquestionable way, but you also have fewer critics who are willing to speak against you openly.

      Or, perhaps they might say, “That witch doctor was no good. We will try the spell again.”
      Perhaps their tradition of trusting elders and shamans improves their ability to oppose invaders, such that communities with that tradition would survive better than others. Perhaps that has been happening for generations.

    • noyann says:

      if you pull out a machine gun and say “Let me demonstrate how physics and chemistry works” and put a few holes in whatever primitive medicine-man is most hostile to you, the tribe will update their beliefs with a real quickness

      What will happen is them thinking “This (wo)man has a much stronger magic — let’s capture them’s fetish with that strong magic!” They will not stop ascribing magic power to things. They may even talk in words of “science” and “technology” and still understand: magic. Cargo cult science.

      • Alex M says:

        So? In that case, just set yourself up as a god and create a religion based around science, so that you can sneak your scientific paradigm into their culture through the back door. If they’re unwilling to change their culture based on the proof that their paradigm is wrong, they’ll change their culture because their new “god” asks them to. And since you’ve just demonstrated that trying to capture somebody with powerful magic (or powerful science) is a move that is unlikely to lead to a long life-expectancy, they’ll generally be inclined to cooperate with you. Make allies with the cooperative tribals, use your allies (equipped with the newfound “magic” you gifted them) to kill off the uncooperative tribals, and you’ve got a pretty solid formula for rapid cultural development. Some people may believe in the magic, some people may believe in the science, but everybody will believe in the smiting. And regardless of the paradigm that they’re operating in, people will want to have the obvious advantages that you can offer them.

        • Simon_Jester says:

          History shows again and again [s]how nature points out the folly of man[/s] how this doesn’t actually work the way you’d think. What actually happens is some combination of:

          1) The individual humans you put in charge of pointing guns at people being, well, human and grotesquely abusing their power in a thousand nasty ways. Which subverts your project in a thousand similarly nasty ways.

          2) Other people not exactly mirroring your thoughts when you think differently and routinely point guns at them. Because what they actually do is say whatever they think you want so that you’ll point the gun away. Just as torture yields not “the correct answer” but rather “whichever answer will make the torture stop,” your approach yields not “the target population becoming open-minded and calmly rational” but rather “the target population making appropriate mouth noises while the first generation of really educated people in that population adopt whichever ideology is most likely to bring about your overthrow, whether it’s true or not.”

          The rapid cultural development tends not to materialize, or rather becomes a total crapshoot, as the real-life history of the non-Western world illustrates.

          In practice, peaceful trade where you don’t shoot anyone who didn’t shoot at you first tends to work about as well and about as quickly. The time lost by waiting for people to decide for themselves to learn the useful parts of your way of life is more than offset by the time gained by not deliberately cultivating a violent, adversarial, and dismissive relationship between you and them.

          • Alex M says:

            You know, you make some really interesting points here, and I’ll have to think about them some more. I could argue that I don’t care about the survival of anybody who stifles scientific advancement, because in both consequentialist and utilitarian terms these people are monsters. Just because there is no direct link to the body count caused by their science denialism doesn’t absolve them of responsibility for the ultimate death toll. Sociology and economics are routinely employed by governments to set policy, and they’re not even real sciences. Somebody needs to be held accountable for that.

            I could also argue that Rome and indeed a lot of other imperialist powers throughout history did an excellent job of forcing “cultural evolution at gunpoint.” And furthermore, that the fear of death is an important driver in forcing dominant elites to update their viewpoints, since it gives them skin in the game which they might not otherwise have.

            That said, I agree that it’s important to quantify the benefits of a fear-based approach versus the drawbacks of the hostility that it incurs. You’ve made a really excellent point here for me to think about.

          • Simon_Jester says:

            @Alex M

            Some additional thoughts:

            I could argue that I don’t care about the survival of anybody who stifles scientific advancement, because in both consequentialist and utilitarian terms these people are monsters.

            Except that your conceptual model seems to be of entire societies of “primitives” who are ‘stifling scientific advancement’ by just… I dunno, passively existing in places you were planning to use for some purpose of your own, I guess? That’s unrealistic.

            In practice, the people who ‘stifle scientific advancement’ and overall inhibit human flourishing tend to be specific subgroups WITHIN a culture. Ruling the overall culture by fear doesn’t necessarily help you, because your need to maintain “iron control” over the culture tends to turn YOU into a group that stifles advancement. Once you have become the colonial overlord in the name of uplifting the primitives, you will rapidly find that he more uplifted the primitives become (i.e. the more exposed they are to complex ideas, advanced technology, and large-scale organization and planning)… Well, the more able they are to use those things against you.

            This is why the university-educated populace of any given colonial dependency tended to be instrumental in bringing about decolonization of that dependency. One of the most common profiles for the leader of a colonial independence movement is “guy who was bright, promising, and knowledgeable, went to school and/or university in the core territories of the colonial overlord, came back angry about the oppression of his people.”

            It’s also why the British rule of India began with the British wrecking large sectors of the Indian domestic economy- to destroy competition for their own export commodities. And why the British organized infrastructure in their dependencies with a focus on things like “ship resources to the sea” and “facilitate troop movement between hot spots” rather than “optimize overall domestic economy.”

            Once you commit yourself to ruling a populace by fear, the necessity of maintaining that rule takes on a life of its own. The people you entrust to be your enforcers will have their own personal goals and desires. And you yourself, and your successors, will find yourselves justifying actions- even actions that undermine the welfare, flourishing, and enlightenment of the populace- in terms of how you need to maintain rule over your chosen group of inferiors for their own good.

            Just because there is no direct link to the body count caused by their science denialism doesn’t absolve them of responsibility for the ultimate death toll. Sociology and economics are routinely employed by governments to set policy, and they’re not even real sciences. Somebody needs to be held accountable for that.

            It can be difficult, from an outside view consistent with being able to practice politics and the art of governance, to differentiate between someone howling for the blood of those who practice “not real science” in the name of “real science” and someone doing the opposite.

            Especially when the person doing the howling dismisses not just specific theories on evidentiary grounds, but entire fields of study. If sociology is “not real science,” then precisely what real science of society exists to be used AS “real science?”

            I could also argue that Rome and indeed a lot of other imperialist powers throughout history did an excellent job of forcing “cultural evolution at gunpoint.”

            A lot of imperialist powers oversaw cultural, technological, and economic progress. A lot oversaw regress. A lot of localized governments oversaw progress, and a lot oversaw regress.

            In large parts of the Mediterranean world, the Romans contributed little or nothing to the state of advancement and scholarship that already existed. They spread a lot of ideas and advancements around widely, and they certainly built a lot of big structures. But aside from the architecture, I would argue that it’s infeasible to disentangle the trends. How do we sort out what was caused by the Romans from what would have happened without Rome?

            And do the Romans get to be credited as an example of “spread civilization at javelin-point,” given that insofar as they did so, they did so by accident? They certainly did not expand their empire for altruistic reasons and had no special desire to “spread civilization?”

            And furthermore, that the fear of death is an important driver in forcing dominant elites to update their viewpoints, since it gives them skin in the game which they might not otherwise have.

            Consider how effective any given method of imposing fear-based controls is at imposing fear on the elite, as opposed to imposing it on the masses.

            Elites tend to remain elites by trimming sails and surviving. The masses have a much longer turnaround time, for a variety of reasons.

            1) The masses of a given society are often trapped in Epistemic Hell. They do not have a reliable set of guidelines for ‘how the world really works’ in their brains, nor do they have easy access to the advice of anyone who does. Even if, hypothetically, YOU know how the world works, you are the terrifying overlord pointing a shotgun at them and demanding that they obey you without question. They cannot exactly quiz you relentlessly. This means that they are in the position of following dictates whose motivation they do not understand, even if those dictates are all objectively correct.

            2) The dictates aren’t usually correct. Almost without exception, some of the things that the self-identified ‘enlightened’ heavily armed side of these disputes wishes to spread are culturally bound. You are not immune to propaganda; there are things you think you ‘know’ on a bone-deep level that are, in fact, the customs of your tribe. Whether it’s “stop eating bugs” or “plant your crops in giant neat rectilinear grids” or “no no, your judicial system should be focusing on retribution and deterrence rather than reparation and restoring community relationships,” everyone who tries this ends up dictating things they don’t really have any objective justification for.

            3) As noted, even if the dictates were all objectively correct, following them instead of the Old Ways would involve expending resources NOT spent on following the Old Ways… and the masses usually have less margin of error. Suppose you are a peasant accustomed to plant wheat the Old Way, and someone tells you at gunpoint to plant potatoes instead. Suddenly you have a problem. You know your family will probably survive if you plant wheat, as they have survived before. You know nothing about potatoes. Even stipulating that the potatoes are an objectively better choice, if something unforeseen goes wrong, it could mean starvation. If something goes wrong with your specific potato crop, you won’t know what to do and the overlords likely won’t tell you very well. And may all the gods help you if the overlords themselves are wrong!

            Thus, the masses are much more likely to resist the New Way of potato planting than the elites, simply because it’s not going to be the elites who will be eating a diet of rats, boot leather, and grass all winter if the potato harvest goes wrong.

    • ec429 says:

      or that economics has almost no ability to predict (while game theory does), and that sociology and economics are not real sciences but just pseudosciences using complex math to masquerade as real sciences

      Macroeconomics can’t predict and is full of bullshit, sure. But micro (aka price theory) predicts just fine. Which really shouldn’t surprise you, because microeconomics is the continuum limit of game theory.

      (I’m not an economist, so you can’t dismiss the above on status-preservation grounds. Even if I were, that argument veers dangerously close to Bulverism.)

    • TheAncientGeeksTAG says:

      SJW’s have an effective weapon: they are able to cancel people. Does that mean they are right, or advancing the truth?

      • Alex M says:

        Oh please. SJWs are going to be extinct by the end of the decade. Cancelling people is only a weapon if your group has enough coordination not to deploy it, and can instead use the credible threat of the weapon as leverage. Since SJWs are constantly jockeying for status between themselves, they have no way to coordinate in such a tight manner – they will always deploy the weapon, regardless of whether it is strategically advantageous to do so. No matter how trivial the offense, some SJW somewhere will find a way to be offended by it. (And even if SJWs as a whole somehow exhibited the self-control to avoid being offended, they can easily be baited into taking offense by cunning trolling – for example, take 4chan’s “It’s OK To Be White” campaign.) What this means is that if you have anything controversial to say, there is never any reason to ally with the Left, because you know that you’re going to be attacked by SJWs regardless and nobody on the Left will have your back. Therefore anybody saying anything remotely controversial will want to ally with the Alt Right, making their camp grow exponentially in numbers and power.

        For example, imagine that somebody perfected the science of game theory to such a degree that they could predict the future. But wait! What if predicting the future caused you to come to the understanding that unrestricted immigration will bring about the collapse of the United States? Do you think the Blue Tribe would ever acknowledge the validity of such a science? Of course not. Somebody would claim that your science is “racist” and would start a cancellation mob. There is no way to prevent it because the Blue Tribe is not coordinated enough to prevent the narcissists and sociopaths on their side (most of whom have gravitated to the SJW camp, since Cancel Culture offers the perfect opportunity to bully people) from forming an online lynch mob. So giving this exciting new science to the Blue Tribe would only result in it being buried and our scientist being attacked. Our hypothetical scientist would be much better off offering his tech boosts to the Red Tribe, since they will form up in lockstep solidarity behind one of their own when he is inevitably attacked by the SJWs.

        Or, let’s say that our hypothetical scientist perfected memetics to the point that he could do mass brainwashing advertisements. Effectively, any politician using these scientific techniques would be unstoppable against a politician who chose not to use them. But wait! Our scientist has committed the sin of being… (gasp) a Straight White Male! Well, in such a case, there’s zero benefit to offering your science to the Blue Tribe. Their SJWs will find an excuse to be “offended” and attack you (since that is simply what narcissistic sociopaths do) and nobody on the Blue Tribe will have your back, since they themselves are all worried about winding up on the receiving end of the circular firing squad. So again, from a game theory perspective, our scientist benefits much more from offering their knowledge to the Red Tribe, because they demonstrate more solidarity and will have his back when he is inevitably attacked.

        Repeat this over multiple iterations, and eventually you will see that the Red Tribe gains access to knowledge, weapons, and allies that the Blue Tribe doesn’t have access to. I suppose that the Blue Tribe will still benefit from the occasional tech boost since there are some people out there who are cringing masochists (aka “good allies”) willing to make the necessary show of deference to SJW bullies, but most people who are both smart enough to develop any kind of advanced science and aggressive enough to weaponize it are unlikely to respond well to bullying. This means that over time, the Red Tribe will gain an unsurpassable advantage. You can see this happening already in Trump’s 2016 victory or the way populism is gaining such a large advantage in the EU. These are not fluke occurrences – they are happening for a reason.

        Eventually, the Blue Tribe will either fold completely (anybody remember the Whig political party) or they will realize that if they want to remain competitive they need to banish the toxic elements within their own ranks. That’s why I can’t see the cult of Social Justice lasting long. They’re already passed their high point, and it’s a long fall down from here.

        • Simon_Jester says:

          Oh please. SJWs are going to be extinct by the end of the decade.

          Interesting prediction you’ve got there. What size hat are you planning to eat if it doesn’t pan out?

          It occurs to me that you’re overlooking something: Red Tribe has its own analogue to the circular firing squad and the more extreme parts of cancel culture.

          What if the hypothetical isolated genius who invents is… a… lady? Or bisexual? Or kind of brownish? Or what if they are none of these things, but just can’t stomach having to abase themselves to an unworthy political Maximum Leader, or having to service changing and shifting values of ‘truth?’

          Besides which, Red Tribe is not exactly great at inviting in people who are not already aligned with it, and has a long history of turning on people who were once their darlings but are now inconvenient to someone who’s too big to fail.

          Consider how the central representative politically active member of Red Tribe would have thought about “the FBI” in 2005 versus 2020, for instance; in just a decade and a half they’ve gone from “steely-eyed guardians of the state against all manner of criminality” to “deep state that’s got it in for Trump and thank God they’re kept restrained by the attorney general he appointed.”

          All in all, well. Revenge fantasies aside, the intellectuals with war-winning ideas might decide they prefer to take their chances with Cancel Culture. Because Red Tribe has been flirting with fascism, and intellectuals don’t fare well under fascism.

          • Aapje says:

            What if the hypothetical isolated genius who invents is… a… lady? […] Or kind of brownish?

            Which party had both the first black secretary of state and the first black female secretary of state?

            Zach Goldberg posted an excerpt from a study recently suggesting that Democrats greatly overestimate the bias among Republicans. For example, they thought that 40% would be willing to vote for a black candidate, but 94% stated that they would do so. They estimated that 43% would be willing to vote for a woman, while 90% said that they would.

            Note that Republicans typically also overestimated the bias, which I attribute to the media being less convincing to Republicans with their focus on the exceptional, but still having a substantial distorting effect.

            Ironically, this probably means that the media is harming the chance of ‘diverse’ candidates, by misinforming people of their electability.

            Bonus: Democrats think that 46% of Republicans mostly agree with white nationalists, but the actual number is about 9%.

            intellectuals don’t fare well under fascism.

            They don’t fare well under Lysenkoism either. Fact is that academia are rapidly losing not just conservatives, but moderates at an equally fast pace. Honeycutt and Freberg’s study suggests that it is radicals of all stripes who can’t help but persecute those with other beliefs, so radical (parts of) academia will probably inevitably discriminate, both by gate keeping and in their work, which logically makes it an enemy to those being discriminated against.

            You are like a person in Nazi Germany who is telling people that Communists are a threat to intellectuals and then cheers on Hitler. Or like a person in Soviet Russia who is telling people that Nazis are a threat to intellectuals and who then cheers on Stalin.

            It’s theoretically correct, but at the same time immensely and utterly incorrect from a pragmatic point of view.

          • Alex M says:

            What if the hypothetical isolated genius who invents is… a… lady? Or bisexual? Or kind of brownish?

            I mean, sure, this could occasionally happen, but statistically they’re most likely to be aspies who don’t fit into society too well. Since these are the people most persecuted by cancel culture, they’re very unlikely to want to side with the Blue Tribe. When it comes to innovation, Aspergers is a superpower, and the Blue Tribe is so obsessed with cancelling people who don’t conform to cultural norms that they’re driving all their geniuses straight into the arms of their enemies.

            Also, while the Red Tribe may traditionally not have been very tolerant, their latest iteration (known to the media as the “Alt Right”) is very diverse and tolerant of anybody, from transgendered people to ethnic minorities. Perhaps the fact that you are obviously a liberal is blinding you to the fact that this is no longer the 1980s and the Republican Party has evolved substantially since then?

            Also, “revenge fantasies aside” is a silly thing to say. Revenge is a fundamental aspect of the human experience. If you are unfairly persecuted by certain elements of society (as many aspies are) it is both natural and proper to seek revenge upon them. This not only makes you feel better, but it serves as a cautionary example to anybody else who might try to persecute your kind. Revenge serves a valuable role in making society function optimally and it shouldn’t be trivialized.

          • Simon_Jester says:

            @Aapje

            Which party had both the first black secretary of state and the first black female secretary of state?

            The second Bush administration represented a different Red Tribe faction that no longer enjoys much support. Remember what I said about how perceptions of, say, the FBI have evolved among central Red Tribe voters in the past fifteen years. There has been a marked political realignment since 2008, because 2008 was a major shock to much of Red Tribe for reasons that ought to be obvious.

            Which party now has sizeable female congressional delegations? And sizeable black caucuses?

            Zach Goldberg posted an excerpt from a study recently suggesting that Democrats greatly overestimate the bias among Republicans. For example, they thought that 40% would be willing to vote for a black candidate, but 94% stated that they would do so. They estimated that 43% would be willing to vote for a woman, while 90% said that they would.

            94% of Republicans say they would be willing to vote for a black candidate.

            And yet when the opposing party elected a black man, considerably more than 6% of Republican voters rapidly swung over to believing a conspiracy theory about the black president secretly being a foreign agent hiding his national origin and/or religion. This was a conspiracy theory that I have a hard time imagining having gotten any traction against a white president.

            The question “would you be willing to vote for a black president” is effectively a proxy for “are you such a stone-cold racial-supremacist that you would refuse to vote for a president of your own party because he was black, and are you prepared to admit it, in a country where this is not a privileged statement under America’s prevailing civic-religious culture?”

            So yeah, only 6% of Republicans say “yes, yes I am totally that racist.” Assuming that some kind of vaguely typical statistical distribution applies to levels of racism among members of a political party, this suggests that we’re looking at the tail of a distribution whose peak is closer to the mean.

            In other words, that there is a rather higher percentage of Republicans who are holding significantly racist views on presidential legitimacy, just not ones intense enough that they would overtly refuse to consider a black president as legitimate, even if he came from their own party and supported them.

            Note that Republicans typically also overestimated the bias, which I attribute to the media being less convincing to Republicans with their focus on the exceptional, but still having a substantial distorting effect.

            Ironically, this probably means that the media is harming the chance of ‘diverse’ candidates, by misinforming people of their electability.

            Tell me, did you seriously think, at any time this century, that anyone other than a straight white male looked likely to win the topmost elected offices in the Republican Party? As in, above the rank of state governor and back-bencher legislator? And not even many of those, in practice.

            Do you think the Republican Party, assuming no major turnover in membership, is likely to nominate a woman or a person of color for the presidency in 2024? What probability would you assign to such an eventuality?

            Or, on a more cynical and simplistic level, I might argue that Republican predictions of what other Republicans as a whole will do may be more accurate than Republican admissions of what they themselves will do.

            Bonus: Democrats think that 46% of Republicans mostly agree with white nationalists, but the actual number is about 9%.

            It depends on what is meant by agreement.

            If you ask me “Hey Simon, what percentage of Republicans will answer ‘yes’ to the question ‘do you mostly agree with white nationalists?’ ”

            Well then, I would like to think I’d predict something like 10%. I am sure I wouldn’t have been surprised to learn that the number is in the neighborhood of 10% or less- which apparently, I just did, I suspect.

            Because directly asking someone “do you mostly agree with white nationalists” is a bit like asking directly asking someone “are you an extremist.” Or, to pick something less political, “are you a liar.” Of course they will probably answer “no.”

            If you ask me “Hey Simon, if the Republican Party leadership started following a white nationalist agenda, about what percentage of Republicans would support said agenda without serious qualms (let alone actually dissenting from and abandoning the party)?”

            Why then, I would reply “somewhere in the neighborhood of 50%, assuming that the usual Party and Party-adjacent institutions did the usual amount of due diligence to promote the freedomizing and totally voluntary self-deportation promotional programs.”

            Either question can reasonably be interpreted as “do you/Republicans support a white nationalist agenda?”

          • Aapje says:

            @Simon_Jester

            Which party now has sizeable female congressional delegations? And sizeable black caucuses?

            That actually strengthens my point. Despite having relatively few women and black people, the party is apparently quite willing to appoint them to high level positions. Ben Carson was the biggest challenger to Trump for a 3.5 month period in the run up to the 2016 Republican primaries & he was appointed to Secretary of Housing and Urban Development by Trump.

            If your actual complaint is not that Republicans rebuff women and black people, but that their platform, campaigning, etc is relatively unattractive to those groups, then I do agree. However, there can be (and in my opinion are) many reasons other than racism and sexism for that.

            And yet when the opposing party elected a black man, considerably more than 6% of Republican voters rapidly swung over to believing a conspiracy theory about the black president secretly being a foreign agent hiding his national origin and/or religion. This was a conspiracy theory that I have a hard time imagining having gotten any traction against a white president.

            Any presidential candidate faces conspiracy theories that partially match some peculiar facts about them. For example, a theory is that Bill and Hillary Clinton killed Epstein. This is not entirely random, because Bill visited Epstein’s Manhattan town house. It also builds upon a longer-running theory that the Clinton’s kill their enemies.

            A candidate not being ‘natural-born citizen’ completely disqualifies them. So it is an extremely attractive attack vector, much more strong than arguing that the candidate is immoral. A concern of a decent number of Republicans is/was ‘anchor babies,’ so the theory also matches a common concern.

            Obama and his close relatives lived a very globalist life, with his mother (Dunham) moving to Hawaii just after it became a state and getting her education there, where she married an already married Kenyan (unbeknownst to her) while she was three months pregnant. Shortly after giving birth to Obama, she moved to Seattle. Then she went back to Hawaii. She divorced Obama Sr, then two years later married an Indonesian man. After graduation, she moved to Indonesia with Obama.

            So Obama and his mom moved around quite a lot. The conspiracy theory ‘merely’ requires that his parents lied about part of that and that they were actually living in Kenya when Obama was born.

            It makes no sense to tell such a narrative about any other candidate, unless they married a foreigner or otherwise had a somewhat plausible reason to be abroad while pregnant.

            BTW, McCain’s natural-bornness was actually also questioned because he was born on a naval base in Panama.

            The question “would you be willing to vote for a black president” is effectively a proxy for “are you such a stone-cold racial-supremacist that you would refuse to vote for a president of your own party because he was black, and are you prepared to admit it

            Sure, that can bias the results one way. On the other hand, how many people will rationalize themselves into voting for such a candidate anyway, if the alternative is someone with politics they hate?

            Assuming that some kind of vaguely typical statistical distribution applies to levels of racism among members of a political party

            Unless this is partially the lizardman constant.

            Tell me, did you seriously think, at any time this century, that anyone other than a straight white male looked likely to win the topmost elected offices in the Republican Party?

            What counts as topmost elected offices?

            The ratio of Democratic vs Republican female senators is 2:1, with the number of female Republican senators being substantially above 0. You’d expect more female Democratic senators purely due to women being more attracted to the Democratic platform, so a gap is not necessarily due to discrimination by voters or such. Any number above 0 shows that female Republicans are not unelectable anyway. Note that the number of female senators seems to have gone up a lot in recent times. The number of women in the senate right now, are nearly half of the total number of female senators, for all of time.

            The ratio of Democratic vs Republican female governers is also 2:1, so similar to the senate.

            The black ratio in the senate is also 2:1, with 1 Republican black senator and 2 Democrats. There are zero black governors.

            Asians are the most lopsided group, with 3 senators vs 0 and 1 governor vs zero (although Democrats ‘cheat’ a bit here because two of the Democrat Asians represent Hawaii, where Asians form the biggest group).

            Both parties have two Hispanic senators and I can’t be arsed to figure which governors are Hispanic.

            Anyway, I don’t see how this disproves other explanations than discrimination. You can easily argue that it (very weakly) suggests the opposite in some cases. For example, with ~90% of blacks voting for the Democrats, you’d expect a 9:1 ratio of black politicians, if black people are also 9 times more likely to become Democratic politicians than Republican politicians.

            Do you think the Republican Party, assuming no major turnover in membership, is likely to nominate a woman or a person of color for the presidency in 2024? What probability would you assign to such an eventuality?

            What is interesting is that the Democrats had way more female and non-white candidates than they have in the senate. Have you considered the possibility that this is due to a strong anti-white male bias at the primary/presidential level (which is quite often openly admitted to) and that you misinterpret a large amount of bias as neutral and much less or no bias as discriminatory?

            Ben Carson’s peak polling in advance of the 2016 Republican primaries was better than the peak polling of any of the non-white male Democratic candidates before or during the 2020 Democrat primaries. I actually think that a populist black and/or female candidate has a huge chance, for various reasons (for example, such a candidate could benefit a lot from free publicity without having to clown it up as much as Trump). In Europe we see quite a few female populist leaders.

            Of course, America is different to most of Europe, with far less opportunity for minority opinions to be represented in politics, although France is somewhat similar to the US in their silencing of minority beliefs and Marine Le Pen (who is quite female) did manage to get into the main presidential election.

            Anyway, putting numbers on this is a bit useless, because it involves predicting a bunch of rare events and uncertain forces that all need to line up. The actual chance for a female Democrat nomination is also hard to predict and depends on such things as otherwise viable female candidates being wise enough to resist outwoking the Democratic voters (unlike Warren), which they are pushed into doing by the out of touch people that tend to surround them.

            If you ask me “Hey Simon, if the Republican Party leadership started following a white nationalist agenda, about what percentage of Republicans would support said agenda without serious qualms (let alone actually dissenting from and abandoning the party)?”

            Why then, I would reply “somewhere in the neighborhood of 50%,

            But how reliable is your belief?

            Polling shows that people also think that they are way more gays, blacks and other minorities than there actually are. Presumably, people are not lying to pollsters about being gay or black, so the far more logical explanation is that the media/movies/etc misinforms people, no?

            Your belief doesn’t seem to have solid proof and it fails occam’s razor in that you need two separate narratives to explain rather similar disparities in two very different ways.

            My explanation actually explains both at once.

            PS. Why 50% and not 20% or 90%?

          • What if the hypothetical isolated genius who invents is… a… lady? Or bisexual? Or kind of brownish?

            Thomas Sowell is more than a little brownish, and Dierdre McCloskey used to be Donald McCloskey. That doesn’t seem to prevent people right of center from viewing them as valuable intellectual allies.

        • Aapje says:

          @Alex M

          The Red Tribe typically folds like a paper towel, though, when something controversial is said and the pitchforks come out. They don’t have people’s back, aside from the issue that they have far less power.

          The rational thing to do is what you see happening, the heterodox abandoning academia or just staying quiet, aside from a few people who try to survive in the wilderness.

          The bigger issue that holds back SJW is that they are usually a small minority and they can be neutered by giving in a bit, virtue signalling, divide and conquer, delay tactics, etc. Also, if they get into power they tend to make such a mess of things that they either destroy what they invade or they get kicked out. To win, you actually need to make institutions that you control work sufficiently.

        • TheAncientGeeksTAG says:

          There’s a general point that having a weapon is not a suffiicient criterion for having the truth, even if the SJW example doens’t pan out specifically. And your disproof of the SJW point is just your own guesswork about what will happen in the future.

  26. Aftagley says:

    I really disagree with this idea, but I’m worried that my reasons for doing so are going to expose some kind of moral or intellectual failings on my part.

    While I don’t think it’s always the best strategy, I’m 100% fine with an using appeal to authority as a way of laying foundational knowledge. For example, take a look at the conversation Scott starts the post with. When the student starts asking about the government mandating no-touching policy, is zhe still working through the process of deciding whether or not the lack of periodic government-mandated quarantines disproves germ theory, or is zhe just, you know, going down a non-critical rabbit hole?

    This is, in general, one of my issues with the Socratic method – it can get you from 0% knowledge to 90% knowledge pretty well, but it really struggles with that last 10% since there are almost always more and more detailed questions you can ask that are all likely critical to a complete knowledge of whatever topic your discussing BUT don’t really matter wrt the fundamentals. If I want a student to know “germs exist, here’s how they spread, here’s some basic precautions you can take” I don’t care if they’ve got a good working understanding of reservoirs.

    Having a good bailout answer of “This question, while interesting, doesn’t really affect your overall understanding of the topic, here are some resources you could consult to learn more, but for now lets move on” is really important both for the teacher, who gets to prioritize their time AND for the student who gets to learn the valuable skill of both accepting information AND understanding they they still have more to learn about the topic.

    • Purplehermann says:

      @Aftagley
      The govt. question is basically “if what you’re saying is true then the consequences don’t work with the rest of what i understand about the world” and this is important, because it is hard to really accept knowledge when it seems to be… wrong.

      So maybe a good rule of thumb is to answer when the info contradicts what the student knows of the world or otherwise the theory simply doesn’t make sense to them.

      If the question is more of a specifics/rabbit hole/but how do you know, appealing to authority and/or telling them to do some research are legitimate recourses if you don’t want to answer

    • albatross11 says:

      The trick here is that as a student learning some new thing you need a mix of:

      a. Intellectual humility–the people studying this have probably thought more about it than you have and probably have more insight.

      b. Skepticism/bullshit detector–the people studying this may have spent their careers making up a huge coherent structure of ideas that doesn’t actually help you understand anything better.

      c. Questioning as a way of making sure you understand the idea. “But then why does X happen” is a good way to help yourself understand the thing you’re being taught better.

      d. Questioning as a way of finding gaps in the idea. “But then why does X happen” might be a lead-in into finding a place where the thing you’re being taught doesn’t explain reality so well.

      Consider someone studying population genetics, macroeconomics, statistics, social psychology, and critical race theory at the same time. I’d say a different mix of a-d is reasonable for each of those, but also that you should be doing all of a-d with each of these subjects.

  27. DinoNerd says:

    *sigh* Both this post and the comment section bring up lots of unpleasant emotions, based on lots of bad memories.

    How often do we actually want the student to understand, rather than to accept what we are telling them on faith? The teacher is probably paid based on the % of their students giving the “right” answer on exams; that answer may be “the Truth according to the ruling ideology”, “the answer that matches what was in the textbook”, “what seems correct if the question is read superficially” or “the best answer available when the test was written, since superseded by later research”, among other things. One of my more useful skills is determining what answer is intended, rather than which one is correct, and holding my nose and giving the intended answer when the two disagree, particularly on any kind of standardized or automated test.

    In several domains of knowledge, that’s arguably the best we can possibly do. There is no objective standard available for judging what is true, beyond “what do I need to claim in order to stop people being mad at me?”

    In many cases where there are objective standards available, the student – and perhaps also the teacher – is not in a position to personally apply them.

    And finally, specific to the example above – do we want the child to understand germ theory, or do we primarily want them to behave in ways advantageous to public health? An awful lot of teachers, and their institutions, pretend to be teaching how things work when their real goal is to influence beahviour.

    It takes a lot of trust, after experiences like these, to believe that it’s safe to visibly prefer understanding the evidence to reciting the correct answer.

    And for the teacher, it’s much the same. They probably expect only their best students to ever actually understand, rather than regurgitate superficially. And even with those students, they probably don’t have time to foster their curiousity, which probably disrupts the basic lesson plan. (They may do so anyway, because it’s one of the few pleasures of the job. But then again, they may not.)

    • Michael Watts says:

      do we want the child to understand germ theory, or do we primarily want them to behave in ways advantageous to public health? An awful lot of teachers, and their institutions, pretend to be teaching how things work when their real goal is to influence behaviour.

      See https://dilbert.com/strip/2006-08-05

    • ksdale says:

      I think this is a really good point. I have a pet theory that goes like this (everything that follows is rank speculation) – many parts of the world have grown immensely in complexity over the past 40 years (tax code, supply chains, all sorts of technology, everyone used to know how to repair their cars and now there are too many electronic components, etc.), and there are lots of people nearing retirement who have personally witnessed most of the complexity spring into existence. They can hold a model of the complexity in their heads, because they witnessed the birth of most of it.

      They are currently training younger people to take their places, but how do we know if those younger people are actually learning how things work, or if they’re just behaving in the correct way(TM)? We won’t know until things start breaking and they need to be fixed by someone who understands how the system works. I think teaching people to give the right answer without understanding builds up a sort of technical debt.

      I do taxes for a living and in my experience, there are two kinds of people who do taxes.

      There are people who take numbers from one place and put them in another place uncritically. And this is fine until the numbers are wrong or in the wrong place, but you have to understand the tax code to know they are wrong and you have to be able to see the wrongness while you’re doing the thing to the numbers.

      And then there are people who largely understand the part of the parts of the tax code they need on a daily basis, and they’re always mentally checking whether the numbers make sense compared to what they represent.

      And the tax code is immensely more complicated than it was 40 years ago. When things go off the rails for the first type of tax preparer, they have *no idea* because they don’t know what the numbers mean. And their clients don’t know, because they have no idea what the numbers mean either, and the IRS never tells them because they actually don’t audit as many people as anyone thinks and they’re mostly focused on the big fish. So stuff just gets done wrong for years!

      If we extrapolate this to the rest of the world, we can assume that all sorts of stuff is getting done wrong all the time and no one knows and everyone thinks they’re way better at their job than they are because they don’t get any negative feedback because the system is too complicated to accurately generate feedback.

      And it might never be a problem, but there also might be a reckoning, where everything breaks in the same way at the same time and we realize that only .1% of the people in a profession know how to fix something, or that the people put in charge of companies and governments will be people who had a “successful” career (defined by never having their lack of understanding rebound on them due to the lack of feedback) and they won’t actually know how the thing works.

      Anyway, rank speculation, like I said.

      • ARod1993 says:

        I’d say I honestly agree with you, and it’s kind of the base of most of the thoughts that keep me up at night: how do you know how good your internal models are, and are you sure that they’re not breaking down in places you don’t notice?

  28. chaosmage says:

    Dude, you’ve built up so much goodwill explaining things to people, you could probably just pull a reverse AMA, an Answer Me Anything, where you just state your confusions, grouped by subject area, and have people compete to give you the best explanation.

    It wouldn’t work for most people, but for you, I think it’s a valid strategy. Or maybe I’m just selfish and think that’d be fascinating to read.

  29. wonderer says:

    But Socrates didn’t ask questions because he really wanted to know the answer. In all of Plato’s dialogues, Socrates asked questions to prove his interlocutors didn’t know what they were talking about, and to advance his own theory of justice/learning/virtue/whatever.

    • The Pachyderminator says:

      He didn’t always have his own theory to advance. In some of the dialogues, he demolishes his interlocutor’s theory – and then stops, leaving everything in confusion.

  30. Michael Watts says:

    I find it funny that you link to “guessing the teacher’s password” here, but when I pointed out that GPT-2 cannot write coherently because it has no model of what it’s saying, your response was to make fun of me for thinking that there’s anything more to communication than choosing words that sound like they might go with the other words.

    • Glenn says:

      I can’t speak for Scott, but: my feeling is that what GPT-2 has might be only quantitatively, rather than qualitatively, different from having a model of what it’s saying (in the sense that you mean). Long before GPT-2, we had simple markov text generators, which literally did just choose each word as a simple function of the few preceding words. But since then, each advance in text generation has come from a system that had a larger, deeper model behind the words than the previous system. For example, if GPT-2 is talking to you about, say, frogs, you will find that unlike a markov generator, it will sometimes be able to stick to the topic of frogs despite a decent amount of non-frog-related content. That doesn’t mean it “understands that the topic is frogs”, but it does have a better model of the conversation than the markov generator does; it at least has something like a notion of topic, whereas the markov generator has only the words, with nothing behind them.

      Of course, the model that is required to produce human-quality textual interaction is a LOT larger and deeper than even GPT-2’s model, and it is by no means a sure thing that a model capable of such an interaction would look all that much like GPT-2. But I suspect that you ultimately believe that such a model is not possible, or that it is only possible in the shape of something that looks exactly like a human brain; whereas I suspect that Scott (like me) believes that it is likely possible with something that looks a lot more like GPT-2 than like a human brain, even if it doesn’t look all that much like either.

  31. Dan L says:

    I feel like this could easily be bundled into a sequence with The Phatic and the Anti-Inductive and Conversation Deliberately Skirts the Border of Incomprehensibility, though ironically they should maybe be read in reverse chronological order. The punchline is that it is not possible for a disclaimer of earnestness to be both useful and low-cost.

  32. blacktrance says:

    A lot of teachers don’t know the answer, and either don’t feel comfortable admitting their ignorance or know that they’re going to lose authority for doing so. If the only difference between your knowledge and mine is that you’ve read the high-school-level textbook already and I haven’t yet, why should I listen to you?

    As for the arrogance police, I think the best approach is to ignore them. Whether you’re asking sincerely or not, you’re questioning something they respect, and a lot of people will take that poorly.

    • Spookykou says:

      why should I listen to you?

      It’s possible that teaching is a skill in and of itself and that teachers can help you learn faster/better than you could on your own even if they do not have any special skill/insights with regard to the subject they are teaching.

    • Simon_Jester says:

      A lot of teachers don’t know the answer, and either don’t feel comfortable admitting their ignorance or know that they’re going to lose authority for doing so. If the only difference between your knowledge and mine is that you’ve read the high-school-level textbook already and I haven’t yet, why should I listen to you?

      Several answers:

      1) Nonrepresentative Sampling
      The high school senior who has read all their textbooks carefully is not the central example of a student in a school. The modal example is an elementary school student, and the median or mean example (agewise) is somewhere in middle school, as a rule. For most of your time in the K-12 school system, a high school graduate who paid attention in all their classes knows a lot more than you do. And after you ascend to the rarefied heights of being credibly able to outperform your teacher, suddenly you go to college and you’re competing with people who have doctoral degrees again.

      Therefore, most of the time. for most individual humans, the thought “what does this old geezer know that I don’t know” is not coming from a place of truth-seeking. It’s coming from the status-hungry part of the monkey brain that wants to somehow become king of the tribe by dethroning the reigning way-too-old monkey and getting to have the biggest share of bananas and fur-grooming attention in geezer-monkey’s place.

      2) Knowing One’s Limitations
      Most people don’t really read a textbook that carefully. Some do, but most don’t. If you can really get all the knowledge the textbook contains just by reading it, and are doing so on schedule ahead of the lessons… well, what’s the point of picking fights with the teacher, you’re already doing great in the class in all probability. If you are, as is more common, often skimming the reading or skipping parts that look unimportant or on days when you’re busy, well… Even if the teacher knows no more than is in the book, the odds are good that if you don’t listen to the teacher, you’ll still know less than them by the end of the year.

      3) Underestimating the Authorities

      You’d be surprised how often the teachers know more than they let on, or more than is superficially present in the book. Sometimes there are practical limits to how much of that knowledge they can convey within the limits of the assigned curriculum and the burden of physically managing the classroom… but they know more than they let on.

  33. benf says:

    As a teacher, it’s important to note that failing a Socratic grilling doesn’t necessarily mean you’re wrong, but it means you’re out of your depth and should try to refer the student to a source where they can get an answer. This, of course, requires you to be able to recognize when you’re out of your depth and not get defensive about it.

    • Viliam says:

      +1

      Also, in extreme cases, there is no source that currently knows the answer. And before that, there is the area where someone probably has the answer, but you have no idea who could it be.

      But saying “I honestly don’t know” is a good start.

      • albatross11 says:

        If you know the subject well enough, you may be able to say “That’s an open question–nobody knows the answer yet.”

  34. EGI says:

    Disclaimer: This reflects mostly experiences in Germany which took place in the nineties, so you may take this with a grain of salt, but just a grain, promise, too much salt is bad for you…

    And the thesis of this post is that you must never, ever say that…

    Oh you sweet summer child…
    No, a thousand times no. I was that kid and, in a way, I still am. Being knocked down for something like this will not make that kid stop questioning, much less noticing the inconsistencies in explanations because, knowing true things and being consistent and sticking to the truth are clearly and self evidently THE most important things in this world. Right?
    So this will just convince the kid that others are stupid and incompetent or evil or most likely both and that it needs to back the next question up with so much incontrovertible evidence that no moron can miss it and no dirty liar can deny it. Then the others will see and stop mocking, right?

    So this kid needs to understand that no matter how right it is (which it will mostly be because it practiced being right for its whole short life and most teachers and other people make lots of mistakes) you do not talk like this in polite company, at least not all the fucking time. Because otherwise you will not become a great scientist but be bullied out of town and become a miserable loner.

    Also the teachers will not be the primary problem. Good and great teachers will typically quickly spot and resolve the mistake, be it theirs or the kids and start to more or less like the kid and less competent ones will either spot it after some back and forth or end the discussion because class needs to go on. The guys who cannot even stand a few pointed questions by a student are luckily not THAT frequent and francly need to find another job.

    One big problem for our kid, mostly in primary school and a few years up, is that the other students will see it frequently and eloquently debating the teacher and, worst of all, being right more often than not, which will induce in many of them the insatiable desire to take this smug guy down a notch. Later on the outright bullying will probably abate, but our kid will still be this tedious know it all who will have no real friends and is only consulted if being right is of critical importance.

    • L (Zero) says:

      I mean, I feel like the actual problem here is that the kid has “no real friends”. And it makes sense that a kid who does not realize that “but this textbook was printed in 2019, is it still true in 2020?” is an annoying waste of time question, will be equally stunted in realizing “do you want to look at the fishes” should not be answered with “the plural is just fish”. But like I think you are not gonna solve “friendship unimportant, know-it-all important” with “know-it-all bad” alone. Isn’t the natural response just “I’m prosecuted by petty teachers AND ignorant jealous peers for my ruthlessness, woe is me”? Maybe you need a more affirmative framework of “friendship good”.

      • Simon_Jester says:

        Yes, but the ability to recognize that one’s pointless nitpicking is an uninteresting and offputting waste of time is still a very important component of the ‘basic social skills’ package for highly intelligent children.

  35. Joshua says:

    In my experience as a teacher, the trick is to remember that when students ask those kinds of questions, it’s because I didn’t anticipate all the potential holes in my explanation. It’s easy to mistakenly think my own explanations are perfect because I already understand the concept I’m trying to explain.

    I try to stay focused on the beauty of seeing the myriad ways an activated student processes information in their efforts to understand. It may seem trite, but those kinds of questions from students reveal to me a kind of failure to fully understand the material in my own part – I get to see all the material I took for granted, and didn’t realize I had to explain to be comprehensive in my teaching.

    I have worked with adult students who had never spent a day in a classroom. I was helping them learn English – and they had never learned to associate sounds with letters (or graphic images). Even their concept of a teacher/student relationship was completely beyond my previous experience. Each day was a humbling lesson for me in all the assumptions I made about teaching and learning that were mostly just a function of my own experience and perspective.

  36. Freddie deBoer says:

    What are the odds that this non-divinely-created universe actually can make sense, can be correctly understood by our immensely limited monkey brains that were not conditioned by evolution to understand the nature of reality?

    • ec429 says:

      I think this is irrelevant; long before we run out of questions answerable by monkey brains, we’ll have engineered our brains to be better. I’m confident that everything worth knowing is rationally knowable, even if some of it takes a few hardware upgrades to fit.

    • Ohforfs says:

      But the universe does not make sense. I mean, at basic physics level, it doesn’t have much sense in it.

  37. Stephen says:

    This reminds me in part of Cunningham’s Law. To the extent that the law is true, it explains why Socratic Grilling is a better method of finding the truth than Socratic Inquiry. People are more motivated to show a smug asshole that they’re wrong. Someone who is genuinely curious merely gets explanations from the good natured; meanwhile a Socratic Grilling gives people the opportunity to show that you’re an idiot.

    • knzhou says:

      Tragedy of the commons, though. As the fraction of smug assholes in a community increases, reasonable people with domain knowledge leave. What you end up with a different set of smug assholes yelling right back at them, but without a deep knowledge of why what they’re saying is right. I’ve been watching this dynamic play out in slow motion on one of my favorite sites, and it’s extremely depressing.

      • Stephen says:

        To clarify, I think this is a problem. I try to foster the kind of community I would want to be a part of by engaging with people who are curious and ignoring the smug assholes.

        Choosing to grill rather than inquire sacrifices conversational norms in favor of answering the immediate question, which is almost never worth it in a world where I have access to modern search engines.

  38. madqualist says:

    Current me would have done OK in this interaction, but 10-years-ago-me wouldn’t have.

    The heuristic that sorts it all out for me isn’t even awareness that the kid isn’t being an ass, but just cultivating skill at being patient, polite, and giving people a lot more benefit of the doubt, for a while, at least.

    So in this situation I’d be thinking “this fuckin’ kid” but politely answering the questions, and by the end I’d have figured out that he was just trying to figure out how things actually work the entire time and nobody would walk away frustrated.

    It’s amazing how many misunderstandings can be avoided and interactions made better by just being patient with people.

    • cuke says:

      I agree with this totally.

      So where does patience come from?

      I don’t know for sure. It seems to me to be a combination of emotional regulation skills (ie, reducing reactivity so one can sit still and listen better and tolerate the imperfection of communication better) and managing expectations (not expecting people to be excellent communicators or to communicate in a way that’s naturally familiar to you, again tolerating the imperfection of communication, humility about one’s own capacities).

      Some of patience also seems to be about awareness — like the capacity for one part of the mind to step back and go, “what story am I telling myself about the conversation I’m having here or about the other person?” or “can I notice where I’m attached to a particular outcome or conclusion and where I’m more open?” or “what attitude am I bringing to this moment generally — tired, irritable, preoccupied about something else, agitated, etc?”

  39. carvenvisage says:

    As good as “in favor of niceness, community, and civilization” is, I think this is an even better mission statement.

  40. I noticed reading this and some of the comments that I must have sidestepped a lot of the status games as a kid by asking questions after class, rather than during it. There are downsides to this – no one else gets to hear the answer, meaning that if it’s informative, only I am benefiting from the after-class exchange – and upsides to this – no one else gets to hear the answer, meaning the teacher doesn’t feel publicly ‘exposed’. (I actually still use the “talk 1:1” tactic online today, if it’s possible/reasonable to, explicitly to give people more room to express themselves casually and not have to worry so much about the public impression they make in the process.)

    I read the exchange as earnestly curious, for the record, but on reflection, I feel this is largely dependent on tone. (Apparently my default tone for observational questions in text form is ‘spoken with wonder and curiosity’; this is interesting to me.) If the student is smirking while asking all those questions, I’d be less inclined to interpret it charitably. If someone is chatting to me and asking me questions, but I’ve met them previously and they’ve a history of being hostile to me, I’ll also be less inclined to interpret it charitably.

    That said, while I tend not to post much on high-traffic sites in general, small counter-point to people suggesting that ‘concern trolls’ should not be fed: Please, please give them proper responses! Or at least don’t insult them or brush them off; properly ignore them if you must. For one, distinguishing between trolls and people with genuine confusion is hard and the very concept of ‘concern troll’ has done a lot of harm to online discourse (I’ve seen enough people I personally knew were making honest enquiries being accused of trolling; it’s very frustrating when people are insulted for trying to understand something better). For two, perhaps far more importantly, remember that public internet debates are less for the benefit of the person you’re directly interacting with, and more for the benefit of everyone else reading the thread silently. Write for those people; make the internet a better place! (Atlas made much of the same point further up in the comments section, with some references. Go check it out!)

    • L (Zero) says:

      All good points! Your second and third paragraphs marry nicely into “distinguishing between trolls and people with genuine confusion is hard” particularly because you may be working with less “tone” and less readily available history. The Asshole Filter is pertinent. A community that is very ready to brand Marxists as trolls, is gonna get flamed a good few times by Marxist trolls and quickly turn away softhearted Marxists who want to be friends.

      Regarding the first paragraph focusing more on the actual teacher-student scenario, there is an auxiliary benefit: If you genuinely have a question that did not come to the mind of the other students because its answer was fairly obvious, you’re not wasting their time during class. Perhaps the teacher’s time is more valuable, even after-hours, but there are plenty of situations I observe where even teachers’ office hours aren’t booked solid, so go for it.

    • MadRocketSci2 says:

      Would that I had an “after class” to ask questions in! (Prior to university.) Our schools had rigid periods, punctuated by bells, an 8 minute dash between rooms, and another period. It was painfully structured and packed. The only free time I had to think (and try to do homework or read) was while everyone else was in the lunch line. (I skipped lunch in highschool to get that little bit of time to myself.)

  41. smaller says:

    Thank you for this wonderful piece. Maybe it’s because I’m hormonal, but I actually started crying while reading this post. In middle school science class, I had a teacher who would yell at me until I started sobbing and then some when I asked questions like the ones the student poses in your scenario. She accused me (and the only other girl who asked questions) of maliciously trying to humiliate her. (She got along better with the boys, so she tolerated a lot more from them. Some of my other teachers were nicer to girls, though; I’m not claiming any systemic sexism–it was just her specifically.)

    At work recently a mentor asked me “Why do you never ask any questions?” and I was just like “eh, I don’t know”, and thought about how they say that growing up means part of you dies. At eight or nine, I had been the child who got kicked out of (fundamentalist, evangelical) Sunday school for asking too many of the wrong questions about theology, much to my dad’s pride, and my mom’s playful annoyance at his inability to hide how proud he was. That little girl would be disappointed in me.

    • Purplehermann says:

      That really sucks
      Hope you feel comfortable being curious and asking questions in the future

  42. skybrian says:

    My usual situation is that I don’t know the answer to [very hard question that nobody has solved, probably] and I’m trying to convince someone that their simple, confidently stated solution has problems. But I don’t know the answer, ether, so I’m just using logic to poke enough holes to show that simple armchair logic can’t solve it and we need data.

    And so, I often find myself often arguing that none of us knows anything. We are not characters in Foundation, psychohistorians don’t actually exist and “calling” one surprising event doesn’t make you one, and the future is genuinely uncertain.

    Does that ever happen to you?

  43. VoiceOfTheVoid says:

    All the “but what if they actually are trying to score points against the teacher?” comments are reminding me just how valuable good-faith discussions are–ones where you can be reasonably certain that your conversational partner is actually trying to learn from you, or have a two-way discussion, rather than trying to prove you wrong or make you look stupid. They’re unfortunately rare on the internet, it seems.

    • L (Zero) says:

      This has definitely been my experience. Environments where you feel supported to talk are extremely important. That can also get wrapped up with tribalist instincts. Some environments and groups can impose a strong sense that your questions or answers won’t be respected.

  44. Reasoner says:

    Great post.

  45. LudwigNagasena says:

    Notice a few things about this conversation.

    The first thing I noticed is that it looks like just a bunch of just so stories with implausible excuses to cover cases that contradict the model, and how unlikely it would look if someone approached with this idea for the first time.

  46. TJ Michael says:

    I think it’s a great metaphor, but sometimes the metaphor is hard to apply to real life.

    In most disscussions it isn’t as easy as saying person A is the teacher and person B is the student. Your friend tells you they won’t vaccinate their kids, are you the student because you’re repeating diffrent questions trying to find the flaws in their model where vaccines are far more harmful than good (and all the conspiratorial layers that then have to be built around that) or are they the student, since they are actaully the ones you are trying to educate?

    What about audience? Students typically outnumber teachers in a teaching environment, however there’s no garuntee for that to be the case in an online environment.

    What about the entitlement to be taught something? A teacher typically has a duty to educate, is that duty transferable to all discussions? Is everyone owned a response and the attention required for socratic grilling? There are plenty of situations where those causing harm without realizing it, grill ernestly to try to understand those who claim they are being harmed by them, do the hurt owe them the explination? What about sanctity, if say I wanted to understand a religion who’s core tennents are kept secret to outsiders, is it my right to keep grilling?

    What if the student’s questions aren’t being effective, not becuase they are annoying or them acting in bad faith? Is the student acting in bad faith because his family taught him not to trust academics, and he disregards any time the teacher cites academic experiments or examples? If the student’s own model is informed by logical fallacies, is it the teacher’s role to engage directly with the student’s questions, or to try and root out those fallacies? What if the teacher suspects the student is curious, but the behaviour the student is modelling for the other students is problematic, IE the student is imapatient, incapable of leaving alone the issue of resiviours and engaging with typical transmission, when resiviours are tommorow’s lesson? What if one of the other students is intentionally confusing others so that they’ll ask these questions and disrupt the class?

    What if the student wants to know more than the teacher is ready or capable of providing at the moment? The newtonian physics model isn’t perfect, a newtonian oriented middle school physics class would come to a standstill if the teacher entertained “endless” grilling, untill the newtonian model would be exhuasted.

    Oh cluck, now I’m doing doing it myself (shrug emoji), asking a bunch of open ended questions, trying to understand how the metaphor works and applies to the discussions I end up having online.

    Did I just write all that in bad faith? I don’t think so, I’m willing to entrtain that since your post made me rexamine my reaction to you previous post, (both here and our exchange on the subreddit), that there’s a natural reaction for self defense there, but also, I don’t think there’s a clear demarkating line between grilling done in bad faith or out of curiousity. It’s sort of that double edged sword of debate I guess, we want to be right, but also to get to the truth. Sure, theres a layer of discussion where we could try to keep away from that, but we can’t always, not when the disscussion leaves the immediate realm of what’s being discussed, and starts leaning on the core values, logical pathways, and philosophical standards / frameworks of those discussing.

    It might seem 2 people can be open minded when discussing the building of a road, but if one is a socialist, and one is a libretarian, how soon before each one will end up defensively justifying their world-view to the other?

    The example assumes the student is acting in good faith, but while there are plenty of examples of people using socrastic grilling in what can clearly be described as bad faith (see Holocaust Deniers), it becomes harder and harder to do so the further the discussion veers into the personal worldview.

  47. kalimac says:

    I get this a lot: I’m told something, I seek understanding by asking a question, I get an assurance that what I was told is true, as if my question were simply a denial of its truth value.

    The other thing I get is being told something so completely out of my understanding that I can’t formulate any questions to clarify it. This may mostly be due to unclear explication, but how would I know? At any rate it leaves the other person at a loss at how to respond. I sympathize, but I can’t help.

  48. kalimac says:

    And another thing I get is when I ask questions to which the answers leave me understanding even less than I did before. Especially when they seem to contradict what I was told earlier. But if I say, “You just said Y; didn’t you say X earlier?” than oh boy watch out. The mere fact of having to ask such a question means that my confusion will never be cleared up. This is almost always due to bad explanations.

    The worst case of this I ever had was when I took a class in home composting. That was the absolute tops in my knowing less when I came out than when I went in.

  49. Jay Searson says:

    I don’t think that the disclaimers are endless or annoying; in fact I find those disclaimers to be one of the best parts of your writing. There are lots of writers out there that don’t let me know that there’s uncertainty in their beliefs or convictions; I appreciate a great deal that you do.

  50. Manx says:

    I agree with your sentiment if you are actually talking about *children* or *students*. However, I can’t help but believe this is a defense of people in our community specifically, and not about children at all. I have found that our community has a lack-of-humility failure-mode when asking unusual questions. There is often a challenging tone, followed by a ‘gotchya’ moment when you don’t answer their weird question immediately or to the extent they desire. Even experts might have to think for a minute about why exactly they do things a particular way, and sometimes the person asking just doesn’t have enough background knowledge to process the answer they were given. I don’t feel it is necessicarily the job of the person being questioned to educate them extensively. Sometimes, ‘I have extensive knowledge in this field and know what I’m talking about – your questions are very amateur and it would take me a long time to explain exactly why your idea is awful- go educate yourself further,’ is entirely warranted. Maybe we should come up with a short-hand for this answer?

    • The Nybbler says:

      Taking one’s own side in a dispute is not a vice.

    • Carl Pham says:

      What I have found useful in this context is to, first, distinguish between those who want a hand-wavy overall rationalization, sort of a Physics For Dummies answer, the kind you get from a pop science article that relies on respect for authority, and those who want an answer they can support themselves — which requires all the much greater labor you describe. If the latter, then giving a popular article answer will indeed be seen as highly dissatisfactory to the interrogator, and for good reasons. In this case, the best thing to do is give them enough to get them to the next level and wish them bon voyage. For example, just reframe their question using the jargon and concepts from the field, so that they can then go off and google those terms and zero in on the learning they need. Or point them towards a Wikipedia article that technically answers the surface question, which you know will lead to further questions, but which will have the plethora of links that allows your interrogator to continue the voyage on his own. Your aim here is not to fully answer the question, but to simply give your interrogator what he needs to take one more step towards the answer.

      The drawback of this is that one does not gain the satisfaction of having answered the question, fully. Still less does one get actual thanks and appreciation, usually. But then, if one’s goal is truly other-centered — you’re really trying to help, and not just feel good about your own expertise — that should not be a serious roadblock.

  51. laretluval says:

    How do you reconcile this with Joe Henrichs’ idea that cumulative cultural evolution is smarter than an individual’s rational analysis?

  52. SCC says:

    Scott, you might be smarter than you think.
    Look at it this way.

    It is easy to surf the internet, if you are someone who went to a “science high school” like Stuyvesant in NYC or TJ in Nova, and who spent all your time on tests or preparing for tests, to come up from underwater once in a while and try to find out who were the lucky people who were the geniuses of their day in the sciences, in philosophy, in math. It certainly was not the people who wrote the AP Calculus course books, one would think ….

    And when such a student tries to find out where mathematical skill has resided, by researching the question on the internet, that student will read thousands of people telling you about how different, say, von Neumann was because “he could multiply six figures times six figures in his head” and he “had brilliant ideas that left everyone in his wake.”

    Well, maybe, maybe not.

    Let’s say you could sing when you were a kid, in tune, and let’s say your teachers hinted that you were a kid wonder, and you went along with that, and one day when you were 10, you heard a really good piece of music by, say, LeClair or Berlioz on the radio when none of your teachers were around, and you researched LeClair and Berlioz, and again and again you read about how they were good but the real geniuses of music was Mozart or Bach.

    Well, maybe, maybe not.

    You see, there were people in Japan, none of whose names almost anyone knows, who were doing physics in just as interesting way as von Neumann in the 1940s – you can look it up – and as far as his great facility with numbers goes, well, any concert pianist can play a thousand notes in succession perfectly in less time than an average conservatory piano teacher can play the same notes with a hundred mistakes.

    And if you listen to a random classical music station for a 24 hour period you will hear so so much music of such transcendent beauty which Mozart, God bless his heart, could not have written.

    What I am saying is, if I have challenged you or anyone else on a specific subject, it was not out of a lack of respect, and I hope you did not think I lacked respect.

    There is not a lot of
    difference between really bright people and the rest of us who discuss the same subject.

    That being said, if anyone does not know the infinite value of being kind to animals or to people who need kindness, well, then, such a person is not really capable of being honestly talked to as an equal.

    And then there is angelic inspiration, but nobody on the “altruistic” side of the libertarian blogosphere ever wants to talk to me about how that has been so important in the history of the world, the history of art, and sometimes in the history of our families and marriages.

  53. hoxy says:

    When I was a kid like 20 years ago I hated raking leaves, and I distinctly remember asking my grandma why we couldn’t just leave them on the lawn. After all, wouldn’t they just decompose and nourish the soil like in a forest? Was I being a little antagonistic? Probably, but I really did wonder about it. Her response was something I don’t quite remember, but it was definitely snarly and dismissive and borderline hateful. The sentiment was something like “No, just shut up and rake the leaves you little shit.” I’ll always remember her as an ignorant and petty household tyrant who was horrifically bad at interacting with her slightly neuro-atypical grandson. Today, lots of organizations really do recommend just letting leaves decompose on your lawn or just mulching them with a lawnmower. But in a way her answer was absolutely right.

    I can’t help but think about all the ways asking too many questions can and will get you in a lot of trouble in the real world – especially with respect to y’know… staying employed. We just don’t live in a culture that values free inquiry, not least of all because odds are when you start pulling threads in a lot of institutions you’re liable to find things that *really don’t make any goddamn sense* yet entire careers, industries, mortgages, and entire lives can depend on everyone just shutting up and affirming what they’re told or at least not rocking the boat.

    There are surely many exceptions, but for the most part academia seems like one of the only places where this kind of interrogating of authority is still even remotely acceptable. I’ll never forget how disturbing and shocking it was during my first big “career” jobs to see just how little anyone actually cared about critically evaluating anything. It was so heartbreaking because I genuinely thought that was the key part of doing anything exceptionally well — asking hard questions, thinking strategically, evaluating assumptions. Nope, it’s often mostly about shutting up and doing what you’re told.

    I don’t think we appreciate how damaging this is for certain types of naturally curious people (especially those that might be a bit neuro-divergent) to have to forcibly shut off and bind up parts of their brain just so they can keep making rent. It reminds me of the cruelty of something like foot-binding, where a natural facet of humanity is painfully contorted and broken up just so it “fits” into some silly little mold demanded by humans cause reasons.

    That’s why I think its imperative to break the stranglehold traditional employment has over so many people’s livelihoods. You’ve never questioned your entire life like you have once you find yourself driving 40 minutes during a ice storm at 7:00 am to get to your office to do work on at a computer — work that could 100% be done online. The gig economy and online freelancing seemed to offer so much promise but good luck qualifying for an apartment or mortgage with sources of income like that. There’s this overwhelming inertia that makes it impossible to get the ball rolling on badly needed reforms like remote working days and shorter work weeks and it probably has something to do with the fact that nobody wants to be the first person to make the suggestion. Try Socratically interrogating your boss about any of that stuff and see how far it gets you.

    Last year I left a decent job writing grants for a nonprofit cultural institution because the very wealthy Board of Directors seemingly had no clue how actual non-profits or grant making worked from the inside. So I’d have to do things like write grant proposals for projects that were actually routine business expenses. If you know anything about foundations these are the absolute last things most of them are interested in funding — so I’d have the stupiying, Sisyphean task of spending days or weeks meticulously compiling attachments and writing proposals for projects I knew with 100% certainty would not be funded. Grants are typically awarded to projects like expanding an after school program, but our board often wanted funding for things like… upgrading our facilities or buying “nicer” chairs for special events. This is very very dumb and borderline unethical. My bosses were nice people, they paid me fairly, but when I tried to gently explain this to them I could see they literally couldn’t comprehend the distinctions, and ultimately they just didn’t care.

    So I left that job thinking the labor market was tight, I had lots of experience, it wouldn’t be too long before I found something else. It took me nearly 6 months to find another job, along the way I fell into an even deeper depression than usual and came closer than ever to killing myself. Today I work for an agency that manages recycling programs and despite the fact that I’m pretty sure a huge percentage of what we do is completely pointless I’m going to keep telling people that the fate of the world depends on fastidiously sorting and rinsing their little plastic containers because I like food and shelter.

  54. Robert Beckman says:

    OK, how could we test that? What would happen if your idea was right? What would happen if mine was right?

    This. The greatest failure of science in recent decades is that science is no longer taught, but rather the church of scientism is proselytized.

    “What do you think you know, and why do you know it” is one of the better summations of the concept, with this quote pointing at the key aspect of science: predicting the future. Science is only about predicting the future, if (criteria A) is met then (outcome B) will occur is the fundamental test of science, and any proposal that can’t be reduced to that form isn’t science, it’s merely a cult.

  55. Purplehermann says:

    I think the sealioning was mostly used to justify people not wanting to explain “obvious” pieces of their worldview, ever.

    You can steelman it, but the sealion folk (the ones who use the term to justify leaving their heads where the sun don’t shine) aren’t worth being charitable to imo

    • Simon_Jester says:

      You might not feel that way if you were on a medical discussion webforum and you kept having to deal with persistent people who keep politely asking why we’re sure homeopathy and the four humors and so on don’t work, and never actually change their minds no matter how many times or how carefully the evidence is laid out.

      But those same people could regroup on the homeopathy website and say “you know, the problem with defenders of allopathic medicine is that they’re so determined to never ever actually defend their own core premises or explain “obvious” parts of their worldview, ever.”

      Good faith debating must necessarily include a willingness to acknowledge when the other party has successfully made a point, and there are a lot of people who wear all the marks of good faith debaters EXCEPT that one.

  56. Galle says:

    That’s one of the biggest problems in human communication, honestly – how do you sort out bad faith actors from good faith ones? How do you tell the difference between someone who’s asking a question because they genuinely want to know the answer and someone who’s asking a question because they think it’s a cheap “gotcha”? And how do you signal that you’re the former instead of the latter.

    One thing that I think can help immensely is what I call “nailing down the goalposts”. This is basically just the old trick of stating your hypothesis in advance, but transferred to the realm of debate. Before asking for evidence, give a hypothetical example of what kind of evidence it would take to convince you. As long as you’re honest about what kind of evidence it would take, this is an easy way to signal genuine curiosity.

    (I’ve also tried to ask people who ask me for evidence to do the same, but that hasn’t worked out so well; I guess you need to be fully immersed in the idea that evidence is about constraining predictions before that idea seems reasonable to you)

    Of course, this does run into the problem that for some ideas the amount of evidence it would take to convince you is monstrously disproportionate. That could make you look close-minded and stubborn, but I think it would still be an improvement.

  57. albatross11 says:

    +1

    Also, I’ll point out that finding places where your current theory doesn’t seem to explain reality is how get a better theory. Kin selection, group selection, and evolutionary game theory all come out of observing stuff that seems really hard to get from {selection, mutation, drift} in an obvious way.

    • Simon_Jester says:

      The trick is that to incrementally improve upon a theory, you generally have to understand the theory.

      There is a significant difference between the kind of conversation had between two experts familiar with a theory, who are poking at its holes, anomalies, and curlicues… And a knowledgeable person familiar with the theory who is being argued at by someone who doesn’t understand the theory but suspects that it is bunk.

  58. Jiro says:

    And the thesis of this post is that you must never, ever say that.

    That’s a terrible thesis.

    What you’ve demonstrated is that you should never say that to a genuinely curious kid.

    In your example the kid is really curious, and of course answering his questions will produce the best result. But that’s because of how you’ve set up the scenario. If you’re faced by actual concern trolls, the result won’t be as good.

    This article is the equivalent of “I have a scenario where a five year old is in the back yard of my house late at night looking in my window, and suspecting him of burglary is clearly bad, so I never, ever should suspect anyone of burglary.”

  59. physticuffs says:

    I’ve been baffled by interpretation differences before. I took an architecture elective in undergrad and one day, the instructor (a professional architect but by no means a professional teacher) was critiquing my design in a fairly jargon-y way and getting extremely annoyed with me when I told her I didn’t understand. I think she assumed I was being willfully ignorant. Eventually, she said something like, “your exterior needs to have a relationship to your interior!” and I told her, “Well, I drew a door here.”

    I truly did not mean it sarcastically. In my “home” department, physics, I’d found that making a statement or asking a semi-rhetorical question that demonstrated *how* I was confused was the fastest way to show my professors exactly what I was struggling with. I’d had tons of conversations with my physics professors that went wonderfully, and never felt that I was addressing them very differently from this architecture professor. I’d never been accused of disrespecting a teacher before. This instructor thought I was so disrespectful that she sent me a furious email the next day, threatening to fail me in the course.

    I took the second semester of architecture out of something like spite. I’m glad I did; it was genuinely very interesting. To this day, I have no fucking clue what it means for your interior and exterior to “have a relationship”.

    • Aapje says:

      Probably that the design of the outside and the inside has to match in style.

    • Spookykou says:

      Off the cuff never having taken an architecture class, they should be related to each other in some way, design, aesthetic, shape, etc, they are two parts of a whole and should not be totally divested or apathetic of one another, although the juxtaposition of an interior and exterior that have no relationship sounds potentially interesting.

  60. Carl Pham says:

    For what it’s worth, I think what you’ve described is not really the sine qua non for being a scientist, at least the modern empiricism-based version, but more a necessity for being a philosopher. It’s the philosopher who is deeply concerned with consistency, and strives to rationalize all the components of a theory. Empirical science — not so much.

    There are a number of king-sized logical inconsistencies in modern physics, for example, and while that is considered regrettable it does not drive intense reasoning effort to resolve them. It’s generally accepted that these result from models which are incomplete, but may well be extremely useful within their known regimes of validity. More importantly, it’s generally understood that the resolution of these inconsistencies will probably not come from a brilliant theoretical insight, but from some new measurement, new data. It’s even accepted that certain logical inconsistencies — wave-particle duality for example — may be here to stay, that there simply *is* no way for the human mind to construct a fully satisfactory mental picture of an underlying reality. What this means is generally considered a philosophical, not a scientific, question.

    I would actually say an intense dislike of logical inconsistency and a drive to rationalize them away is a bit of a drawback to becoming an empirical scientist. It overweights the role of reasoning and underweights the importance of measurement, and makes it difficult to be in a state of “suspended disbelief” and openness to *any* result which is best for making good empirical observations. It is *extremely* hard for the ordinary human being to not let his preconceptions about how things “ought” to work — his rational worldview — to influence, subtly or otherwise, his observations — his “translations” from pure sensory data into the memes, trends, perceptions of experimental error, level of “interesting” or “not interesting” that constitute the results from experimental observation. The best scientists have the ability to be have somewhat of a Zen blank mind during this process, record everything without bias or preconception, and be completely open to the bizarre, the tiny deviation, the unexpected. Which is where genuine scientific advance (in the empirical model) comes from.

    There are of course exceptions — situations in which niggling logical inconsistencies have proved to be the loose thread which a brilliant theoretician seized on to create whole new ways of understanding reality. Relativity is the classical (so to speak) example. But I would argue this is the exception, not the rule. Most genuine scientific advance occurs from really superior observations, and these come most often from people with a profound distrust of the power of logical reasoning, who are untroubled by apparent inconsistencies and logical lacunae, and who have the ability to “expect nothing” when they observe.

    Of course, one also has to be curious, too. And not especially respectful of authority. And both of *those* character traits are also part of the nature of your hypothetical student. So I would certainly agree the best teachers (of objective subjects) do not discourage curiosity and are not offended by a lack of respect for authority. But for *science* the best teachers also warn the student against too great a reliance on human reasoning, on pure logic, since the human mind is endlessly capable of rationalization, and the apparent soundness of any line of reasoning is unfortunately a completely unreliable guide to the correctness of its conclusions.

  61. Robert Jones says:

    When I left school, one of my teachers expressed the hope that I would be as much trouble for my university tutors as I had been for them, which I think is because I asked a lot of these types of questions. It only once got me into trouble, when the headmaster referred to Einstein as a mathematician and I said I thought he was a physicist (which in retrospect I can see was irrelevant to the point being made) and he accused me of impudence.

    My teachers may well have been unusually patient, but these questions do fit better in the teacher-student relationship than they do elsewhere, and as an adult I’ve learnt to keep at least some of my questions to myself. It’s just not polite to grill everyone. Between pupils and masters, there is no status game to be played, and everybody involved understands that the questions are intended to get at the answers. This just doesn’t work as well in interactions between adults, because the question is more likely to be perceived as an attack on the other’s status.

    • Between pupils and masters, there is no status game to be played

      I disagree. If the pupil has in fact found an error by the master, the master may well feel he will lose status by admitting it. I’m thinking of a real world example — I was the student — where a physics teacher in a very good high school refused to either admit that his answer to a problem was provably wrong or defend it, and I’m pretty sure that was the reason.

  62. MadRocketSci2 says:

    I was a bit of an annoying kid, and tended to ask a lot of questions. For me it 100% was trying to build a model of the world, and pretty much no “social monkey games”, because I was more or less oblivious at the time that that was even a thing. I had an *intense* drive to understand, and not a lot of understanding: not being able to model the world was painful to me.

    I did have a lot of trouble with teachers: For the vast majority of them, it was simply because they had finite time (I had infinite questions) and other students to attend to. One or two of them reacted to me as if I were being insolent, and one of them (my 2nd grade teacher) actually tried to have me removed from school for being psychologically unfit. I recall an incident where she cut up my school library card, because the last thing someone like me needed was more encouragement with other sources of information.

    After elementary school, none of this was a problem. In college, my professors actually liked this sort of interaction. One extremely eccentric angry east-european professor didn’t like questions (he would ask if there were any questions at the end of the class as a rote mannerism, then explode in fury if there were, because obviously we were being obtuse): By that time I had the maturity to understand (or at least handle) other people and their reactions, and avoid being an eccentric pest myself.

    ****

    PS: I have more sympathy and understanding for my poor teachers now that the shoe has been on the other foot a few times. It takes a lot of time and effort to drudge up all the pieces from memory to rebuild foundational knowledge so that someone else can get it.

  63. stephentrobbins says:

    Dear Scott
    I don’t find your disclaimers annoying and sometimes they both save me the trouble of thinking about disclaimers myself and remind me to consider how narrowly or broadly applicable the content might be.

  64. DPiepgrass says:

    I once had a great linguistics professor, so I asked her a lot of questions… and she got upset. She thought I was questioning her competence, so I had to explain that actually I was asking her questions because I thought she could answer them. I explained that I didn’t bother to ask questions to most other professors because I didn’t think they were good enough to answer the questions properly. She was, in fact, my favorite professor in university, and eventually I bought her a gift basket to show my appreciation.

  65. Douglas Knight says:

    This really whitewashes schools.

  66. omegaile says:

    Jesus may have died for our sins, but Socrates died for our right to question. Don’t let his death be in vain.